Maternity Exam #3 questions

Ace your homework & exams now with Quizwiz!

18.The nurse understands that the classic symptom of endometritis in a postpartum client is which of the following? A) Purulent, foul-smelling lochia B) Decreased blood pressure C) Flank pain D) Breast is hot and swollen

Answer: A Explanation: A) Assessment findings consistent with endometritis are foul-smelling lochia, fever, uterine tenderness on palpation, lower abdominal pain, tachycardia, and chills. B) Decreased blood pressure is a sign of hemorrhage. C) Flank pain is a symptom of a urinary tract infection. D) The breast being hot and swollen is a symptom of engorgement.

1) The nurse is caring for the newborn of a diabetic mother whose blood glucose level is 39 mg/dL. What should the nurse include in the plan of care for this newborn? A) Offer early feedings with formula or breast milk. B) Provide glucose water exclusively. C) Evaluate blood glucose levels at 12 hours after birth. D) Assess for hyperthermia.

Answer: A Explanation: A) IDMs whose serum glucose falls below 40 mg/dL should have early feedings with formula or breast milk (colostrum). B) If normal glucose levels cannot be maintained with oral feeding, an intravenous (IV) infusion of glucose will be necessary. C) Blood glucose determinations should be performed by heel stick hourly during the first 4 hours after birth and at 4-hour intervals until the risk period (about 48 hours) has passed. D) Hypothermia is a potential problem for the SGA newborn due to decreased brown fat stores and minimal subcutaneous tissues.

6) In planning care for a new family immediately after birth, which procedure would the nurse most likely withhold for 1 hour to allow time for the family to bond with the newborn? A) Eye prophylaxis medication B) Drying the newborn C) Vital signs D) Vitamin K injection

Answer: A Explanation: A) Eye prophylaxis medication instillation may be delayed up to 1 hour after birth to allow eye contact during parent-newborn bonding. B) Drying the newborn after birth is an essential nursing intervention, and should not be withheld. C) Taking vital signs is an essential nursing intervention, and should not be withheld. D) Vitamin K usually is given within 1 hour following birth, but does not interfere with eye contact and bonding between parent and newborn.

16)Benefits of skin-to-skin care as a developmental intervention include which of the following? Note: Credit will be given only if all correct choices and no incorrect choices are selected. Select all that apply. A) Routine discharge B) Stabilization of vital signs C) Increased periods of awake-alert state D) Decline in episodes of apnea and bradycardia E) Increased growth parameters

Answer: B, D, E Explanation: A) Early discharge is a benefit of skin-to-skin care as a developmental intervention. B) Stabilization of vital signs is a benefit of skin-to-skin care as a developmental intervention. C) Increased periods of quiet sleep is a benefit of skin-to-skin care as a developmental intervention. D) A decline in episodes of apnea and bradycardia is a benefit of skin-to-skin care as a developmental intervention. E) Increased growth parameters are a benefit of skin-to-skin care as a developmental intervention.

1) The nurse prepares to admit to the nursery a newborn whose mother had meconium-stained amniotic fluid. The nurse knows this newborn might require which of the following? A) Initial resuscitation B) Vigorous stimulation at birth C) Phototherapy immediately D) An initial feeding of iron-enriched formula

Answer: A Explanation: A) The presence of meconium in the amniotic fluid indicates that the fetus may be suffering from asphyxia. Meconium-stained newborns or newborns who have aspirated particulate meconium often have respiratory depression at birth and require resuscitation to establish adequate respiratory effort. B) Stimulation at birth should be avoided to minimize respiratory movements. C) Phototherapy is not required immediately. D) Mild or chronic anemia in an infant may be treated adequately with iron supplements alone or with iron-fortified formulas.

8) The nurse assesses the newborn's ears to be parallel to the outer and inner canthus of the eye. The nurse documents this finding to be which of the following? A) A normal position B) A possible chromosomal abnormality C) Facial paralysis D) Prematurity

Answer: A Explanation: A) The top of the ear (pinna) is parallel to the outer and inner canthus of the eye in the normal newborn. B) Low-set ears could indicate a chromosomal abnormality. C) This ear position is not indicative of facial paralysis. D) This ear position is not indicative of prematurity.

17)In caring for the premature newborn, the nurse must assess hydration status continually. Assessment parameters should include which of the following? Note: Credit will be given only if all correct choices and no incorrect choices are selected. Select all that apply. A) Volume of urine output B) Weight C) Blood pH D) Head circumference E) Bowel sounds

Answer: A, B Explanation: A) In order to assess hydration status, volume of urine output must be evaluated. B) In order to assess hydration status, the infant's weight must be evaluated. C) Blood pH is not an indicator of hydration. D) Head circumference is not an indicator of hydration. E) Bowel sounds are not an indicator of hydration.

31.Clinical features of posttraumatic stress disorder (PTSD) include which of the following? A) Difficulty sleeping B) Acute awareness C) Flashbacks D) The need to be constantly around others E) Irritability

Answer: A, C, E Explanation: A) A clinical feature of PTSD is difficulty sleeping. B) Numbness, not acute awareness, is a clinical feature of PTSD. C) A clinical feature of PTSD is intrusive thoughts and flashbacks to the threatening event. D) Avoidance of others is a clinical feature of PTSD. E) A clinical feature of PTSD is irritability.

18)To promote infant security in the hospital, the nurse instructs the parents of a newborn to do which of the following? A) Keep the baby in the room at all times. B) Check the identification of all personnel who transport the newborn. C) Place a "No Visitors" sign on the door. D) Keep the baby in the nursery at all times

Answer: B Explanation: A) Newborns need to return to the nursery at times. B) Parents should be instructed to allow only people with proper birthing unit identification to remove the baby from the room. If parents do not know the staff person, they should call the nurse for assistance. C) A "No Visitors" sign would not ensure safety. D) Newborns will need to return to the nursery at times, but the newborn is not required to be there at all times.

6) The nurse is assessing a 2-hour-old newborn delivered by cesarean at 38 weeks. The amniotic fluid was clear. The mother had preeclampsia. The newborn has a respiratory rate of 80, is grunting, and has nasal flaring. What is the most likely cause of this infant's condition? A) Meconium aspiration syndrome B) Transient tachypnea of the newborn C) Respiratory distress syndrome D) Prematurity of the neonate

Answer: B Explanation: A) There was no meconium in the amniotic fluid, which rules out meconium aspiration syndrome. B) The infant is term and was born by cesarean, and is most likely experiencing transient tachypnea of the newborn. C) The infant is not premature and therefore is not likely to be experiencing respiratory distress syndrome. D) The infant is not premature.

24)The nurse determines the gestational age of an infant to be 40 weeks. Which characteristics are most likely to be observed? Note: Credit will be given only if all correct choices and no incorrect choices are selected. Select all that apply. A) Lanugo abundant over shoulders and back B) Plantar creases over entire sole C) Pinna of ear springs back slowly when folded. D) Vernix well distributed over entire body E) Testes are pendulous, and the scrotum has deep rugae

Answer: B, E Explanation: A) Vernix and lanugo both disappear as the infant reaches term. B) Sole (plantar) creases are reliable indicators of gestational age in the first 12 hours of life. C) The pinna's springing back slowly indicates prematurity. By term, the newborn's pinna is firm, stands away from the head, and springs back quickly from the folding. D) Vernix and lanugo both disappear as the infant reaches term. E) By term, the testes are generally in the lower scrotum, which is pendulous and covered with rugae.

10)The nurse has received the shift change report on infants born within the previous 4 hours. Which newborn should the nurse see first? A) 37-week male, respiratory rate 45 B) 8 pound 1 ounce female, pulse 150 C) Term male, nasal flaring D) 4-hour-old female who has not voided

Answer: C Explanation: A) A normal respiratory rate is 30 to 60 breaths/min. This infant has no unexpected findings. B) A normal pulse is 110 to 160 beats/min. This infant has no unexpected findings. C) Nasal flaring is an indication of respiratory distress. The nurse must be immediately available to provide appropriate interventions for a newborn in distress. D) The first voiding should occur within 24 hours and first passage of stool within 48 hours. This is not a life-threatening condition.

14)The nurse is making an initial assessment of the newborn. The findings include a chest circumference of 32.5 cm and a head circumference of 33.5 cm. Based on these findings, which action should the nurse take first? A) Notify the physician. B) Elevate the newborn's head. C) Document the findings in the chart. D) Assess for hypothermia immediately.

Answer: C Explanation: A) A physician would be notified only if findings were abnormal. B) There is no indication that the newborn's head should be elevated. C) Documentation is the appropriate first step. The average circumference of the head at birth is 32 to 37 cm, and average chest circumference ranges from 30 to 35 cm. D) None of the findings indicates hypothermia. No data on temperature are given

25)Which is the criterion that practitioners use to diagnosis fetal demise in utero? A) Absence of fetal movement B) Absence of fetal heartbeat C) Increased serum estriol D) Increased serum testosterone

Answer:B Explanation: A) While the absence of fetal movement often cues the pregnant client that there may be something wrong, this is not the criterion for the diagnosis of fetal demise in utero. B) Absence of a fetal heartbeat is the criterion that practitioners use to diagnose fetal demise in utero. C) A decreased, not increased, level of estriol that accompanies fetal demise will often initiate labor. However, this is not the criterion for diagnosis of fetal demise in utero. D) Testosterone levels do not cause, nor do they change, as a result of fetal demise; therefore, this is not a criterion used to diagnose fetal demise in utero

28)The special care nursery nurse is working with parents of a 3-day-old infant who was born with myelomeningocele and has developed an infection. Which statement from the mother is unexpected? A) "If I had taken better care of myself, this wouldn't have happened." B) "I've been sleeping very well since I had the baby." C) "This is probably the doctor's fault." D) "If I hadn't seen our baby's birth, I wouldn't believe she is ours."

Answer: B Explanation: A) Some parents may feel guilty about their baby's condition and think they have caused the problem. B) A sick infant is a source of great anxiety for parents. This response is from the mother would be unexpected. C) Parents express grief as shock and disbelief, denial of reality, anger toward self and others, guilt, blame, and concern for the future. D) Parents express grief as shock and disbelief, denial of reality, anger toward self and others, guilt, blame, and concern for the future.

13)The nursing instructor is conducting a class about attachment behaviors. Which statement by a student indicates the need for further instruction? A) "The en face position promotes bonding and attachment." B) "Ideally, initial skin-to-skin contact occurs after the baby has been assessed and bathed." C) "In reciprocity, the interaction of mother and infant is mutually satisfying and synchronous." D) "The needs of the mother and of her infant are balanced during the phase of mutual regulation."

Answer: B Explanation: A) The mother has direct face-to-face and eye-to-eye contact in the en face position, which is crucial to attachment. B) Ideally, initial skin-to-skin contact is immediate. The benefits of this practice are supported by a preponderance of evidence. C) Reciprocity is an interactional cycle that occurs simultaneously between mother and infant. It involves mutual cuing behaviors, expectancy, rhythmicity, and synchrony. D) In the phase of mutual regulation, mother and infant seek to deal with the degree of control to be exerted by each in their relationship.

16)At birth, an infant weighed 8 pounds 4 ounces. Three days later, the newborn is being discharged. The parents note that the baby now weighs 7 pounds 15 ounces. The nurse explains that the change in the newborn's weight is which of the following? A) Excessive B) Within normal limits C) Less than expected D) Unusual

Answer: B Explanation: A) This is not an excessive weight loss. B) This newborn's weight loss is within normal limits. A weight loss of up to 10% for term newborns is considered within normal limits during the first week of life. C) This is not a less-than-expected amount of weight loss. D) This weight loss is not unusual.

22)The nurse is assessing a drug-dependent newborn. Which symptom would require further assessment by the nurse? A) Occasional watery stools B) Spitting up after feeding C) Jitteriness and irritability D) Nasal stuffiness

Answer: C Explanation: A) An occasional watery stool can be associated with the normal newborn. B) Spitting up after some feedings can be associated with the normal newborn. C) Jitteriness and irritability can be an indicator of drug withdrawal. D) Nasal stuffiness can be associated with the normal newborn

38)The nurse is preparing to gavage-feed a preterm infant. Put the steps in the order in which the nurse should provide this feeding. 1. Check pH of the gastric aspirate 2. Elevate the syringe 6-8 inches above the infant's head 3. Measure from the tip of the nose to the earlobe to the xiphoid process 4. Clear the tubing with 2-3 mL of air 5. Lubricate the tube by dipping it into sterile water

Answer: 3, 5, 1, 2, 4 Explanation: Measurement occurs before inserting the tube into the infant. Lubricating the tube helps with passage into the infant. After passage, the pH of gastric contents is determined. The syringe is elevated above the infant's head for the feeding. At the end of the feeding the tube is cleared with 2-3 mL of air

17.The postpartum client states that she doesn't understand why she can't enjoy being with her baby. What would the nurse be concerned about? A) Postpartum psychosis B) Postpartum infection C) Postpartum depression D) Postpartum blues

Answer: C Explanation: A) Postpartum psychosis is more severe, and includes hallucinations and irrationality, which are not represented in this situation. B) Postpartum infection is not related to this situation. C) Postpartum depression can impair maternal-infant bonding and can cause developmental and cognitive delays in the child. D) Postpartum blues episodes occur frequently in the week after birth, are associated with hormonal fluctuations, and are typically transient.

7. A postpartum client reports sharp, shooting pains in her nipple during breastfeeding and flaky, itchy skin on her breasts. Which of the following does the nurse suspect? A) Nipple soreness B) Engorgement C) Mastitis D) Letdown reflex

Answer: C Explanation: A) These are not symptoms of nipple soreness. B) These are not symptoms of engorgement. C) Signs of mastitis include late-onset nipple pain, followed by shooting pain between feedings, often radiating to the chest wall. Eventually, the skin of the affected breast may become pink, flaking, and pruritic. D) These are not symptoms of the letdown reflex.

15)The neonatal special care unit nurse is overseeing the care provided by a nurse new to the unit. Which action requires immediate intervention? A) The new nurse holds the infant after giving a gavage feeding. B) The new nurse provides skin-to-skin care. C) The new nurse provides care when the baby is awake. D) The new nurse gives the feeding with room-temperature formula.

Answer: D Explanation: A) If the infant cannot be held during a feeding, she should be held after feedings for comfort. B) Skin-to-skin (kangaroo) care has become the norm in NICUs across the United States and is defined as the practice of holding infants skin to skin next to their parents. C) Preterm babies spend more time in sleep cycles; it is best to not interrupt sleep when possible. D) Preterm babies have little subcutaneous fat, and do not maintain their body temperature well. Formula should be warmed prior to feedings to help the baby maintain its temperature.

22)A postpartum client calls the nursery to report that her 3-day-old newborn has passed a green stool. What is the nurse's best response? A) "Take your newborn to the pediatrician." B) "There might be a possible food allergy." C) "Your newborn has diarrhea." D) "This is a normal occurrence."

Answer: D Explanation: A) It is not necessary for the client to take her newborn to the pediatrician. B) The green color of stool is not due to food allergies. C) The green color of stool is not due to diarrhea. D) The newborn's stools change from meconium (thick, tarry, black) to transitional stools (thinner, brown to

7) A new parent reports to the nurse that the baby looks cross-eyed several times a day. The nurse teaches the parents that this finding should resolve in how long? A) 2 months B) 2 weeks C) 1 year D) 4 months

Answer: D Explanation: A) It usually takes a little longer than 2 months for transient strabismus to disappear. B) Transient strabismus lasts longer than 2 weeks. C) Transient strabismus generally does not last 1 year. D) Transient strabismus is caused by poor neuromuscular control of the eye muscles and gradually regresses in 3 to 4 months.

8) The nurse caring for a postterm newborn would not perform what intervention? A) Providing warmth B) Frequently monitoring blood glucose C) Observing respiratory status D) Restricting breastfeeding

Answer: D Explanation: A) Provision of warmth is an important intervention for postterm newborns. B) Frequent monitoring of blood glucose is an important intervention for postterm newborns. C) Observation of respiratory status is an important intervention for postterm newborns. D) Breastfeeding is an appropriate means of feeding for the postterm newborn

32.A patient who is hemorrhaging after a vaginal delivery is being considered for a uterine tamponade. What should the nurse instruct the patient about this process? Note: Credit will be given only if all correct choices and no incorrect choices are selected. Select all that apply. 1. A balloon is inserted into the uterus 2. The balloon is kept in place for 12 hours 3. The balloon is inflated with 300 to 500 mL of saline 4. After removal, the uterus is packed with sterile gauze 5. The tube has an open tip to permit bleeding to be visualized

Answer: 1, 3, 5 Explanation: If utero-tonic agents are unsuccessful at correcting uterine atony, the physician may use uterine tamponade. One technique of uterine tamponade is the use of the Bakri balloon, which is inserted into the uterine cavity and inflated with 300-500 mL of isotonic saline to provide pressure against the uterine walls. The tube has an open tip, which permits any continuous bleeding from the uterus to be visualized. If bleeding is controlled, the tamponade is removed after 24 hours and not 12 hours. Packing the uterus with sterile gauze is no longer favored as a method of tamponade.

34)The nurse is preparing to provide a newborn with an injection of vitamin K. In which order should the nurse complete the following steps? 1. Cleanse skin with alcohol and allow to dry 2. Aspirate and then inject the medication slowly 3. Insert a 25-gauge 5/8 inch needle at a 90 degree angle 4. Remove the needle and massage with an alcohol swab 5. Bunch skin over mid-anterior lateral aspect of the thigh

Answer: 1, 5, 3, 2, 4 Explanation: Procedure for vitamin K injection. Cleanse area thoroughly with alcohol swab and allow skin to dry. Bunch the tissue of the mid-anterior lateral aspect of the thigh (vastus lateralis muscle) and quickly insert a 25-gauge 5/8-inch needle at a 90-degree angle to the thigh. Aspirate, and then slowly inject the solution to distribute the medication evenly and minimize the baby's discomfort. Remove the needle and massage the site with an alcohol swab.

36.A postpartum patient weighing 165 lb is prescribed a subcutaneous injection of Enoxaparin 1 mg/kg twice daily. The medication available is 50 mg/mL. How many mL of medication should the nurse provide for each injection? (Calculate to the nearest tenth decimal point.)

Answer: 1.5 mL Explanation: First determine the patient's weight in kg by dividing 165 lb by 2.2 or 165/2.2 = 75 kg. Next determine the prescribed dose by multiplying the amount of 1 mg by kg of body weight or 1 mg × 75 kg = 75 mg. Next calculate the amount of medication to provide by using the equation Dosage Required/Dosage Available × mL; 75 mg/50 mg × 1 mL = 75/50 × 1 = 1.5 mL. The nurse should provide 1.5 mL for each injection.

39)During a home visit the mother of a 2 week old newborn is concerned that the baby always seems to be "wet" and wonders if this is normal. The newborn weighs 4 kg. How many mL of fluid should the nurse explain that the infant makes each day? (Round to the nearest whole number.)

Answer: 100 mL Explanation: The newborn voids 5 to 25 times every 24 hours, with a volume of 25 mL/kg/day. If the newborn weighs 4 kg then the amount of urine produced every day is 25 mL × 4 = 100 mL.

38)At birth, a newborn weighs 8 pounds 4 ounces. When discussing the infant's weight over the next week, what is the maximum amount of weight the mother should expect that the infant will lose? (Calculate to the first decimal point.)

Answer: 13.2 ounces Explanation: The maximum amount of weight loss over the first 5 to 10 days of extrauterine life is 10%. If the infant weighs 8 pounds 4 ounces, this weight in ounces is 132 ounces. Multiply this weight by 10% to find that the mother can expect the infant to lose 13.2 ounces. When converted to pounds and ounces, the infant should not weight less than 7 pounds 5 ounces.

37)The nurse is preparing to instruct the parents of a newborn on the care of the umbilical cord. In which order should the nurse provide these instructions? 1. Check the cord for color 2. Wash hands with soap and water 3. Fold diaper below umbilical cord 4. Clean cord and base of cord with cotton swab 5. Check the cord for odor or oozing of green material

Answer: 2, 4, 1, 5, 3 Explanation: Wash hands with clean water and soap before and after care. Clean cord and skin around base with a cotton swab or cotton ball. Clean 2 to 3 times a day, or with each diaper change. Cord should look dark and dry up before falling off. Check cord each day for any odor, oozing of greenish yellow material, or reddened areas around the cord. Fold diapers below umbilical cord to air-dry the cord.

34.A patient experiencing postpartum hemorrhage is prescribed to receive 741 mL of a crystalloid solution. How many mL of blood did this patient lose from the hemorrhage? (Calculate to the nearest whole number.)

Answer: 247 mL Explanation: Lost intravascular volume is replaced initially with rapid administration of warmed crystalloid solutions, in a 3 mL solution per 1 mL of estimated blood lost ratio. To determine blood loss, divide the volume of fluid by 3 or 741/3 = 247 mL.

38)At birth a newborn weighed 7 pounds 10 ounces. If the average weight gain is 7 ounces every week for the first 6 months, what weight should the nurse expect when assessing an infant that is 20 weeks old? (Calculate the weight in ounces.)

Answer: 260 ounces Explanation: Multiply 7 ounces × 20 = 140 ounces. Divide the amount in ounces by 16 or 140/16 = 8.75. Convert 8.75 to pounds and ounces by using the equation 75/100 = x/16; 1200 = 100x; x= 12 ounces. Convert the birth weight to ounces: 7 × 16 = 112 + 10 = 122. Convert the weight gained over 20 weeks: 8 × 16 = 128 + 12 = 140. Add the two weights: 122 + 140 = 260. The baby should weigh 260 ounces by 20 weeks.

37)At birth a newborn's head circumference is 13 inches. What should the nurse expect the chest circumference to be in cm? (Round to the nearest whole number.)

Answer: 31 cm Explanation: The circumference of the newborn's head is approximately 2 cm greater than the circumference of the newborn's chest at birth. First determine the infant's head circumference in cm by multiplying 13 inches by 2.54 cm or 13 × 2.54 = 33.02 or 33 cm. If the chest circumference is 2 cm smaller than the head circumference, the nurse should expect the infant's chest to measure 31 cm.

23.The nurse suspects that a client has developed a perineal hematoma. What assessment findings would lead the nurse to this conclusion? A) Facial petechiae B) Large, soft hemorrhoids C) Tense tissues with severe pain D) Elevated temperature

Answer: C Explanation: A) Facial petechiae do not indicate perineal hematoma. B) Large, soft hemorrhoids are not indicative of perineal hematoma. C) Tenseness of tissues that overlie the hematoma is characteristic of perineal hematomas. D) An elevated temperature is not a symptom of a perineal hematoma.

38)Put the following components specific to a postpartum examination in the proper sequential order: 1. L-lochia 2. E-emotional 3. H-Homans'/hemorrhoids 4. B-breasts 5. E-episiotomy/lacerations

Answer: 4, 1, 5, 3, 2 Explanation: If following BUBBLEHE (B: breast, U: uterus, B: bladder, B: bowel, L: lochia, E: episiotomy/laceration, H: Homans'/hemorrhoids, and E: emotion) the breasts should be assessed first followed by lochia, episiotomy, Homan's/hemorrhoids, and ending with emotion

38)The mother of a ne wly circumcised infant is concerned about caring for the infant at home. What should the nurse instruct the mother about the infant's care? Place the following actions in the order that should be instructed to the mother. 1. Patdry 2. Rinse area with warm water 3. Fasten diaper snuggly over the penis 4. Apply small amount of petroleum jelly 5. Squeeze water over the circumcision

Answer: 5, 2, 1, 4, 3 Explanation: Squeeze water over circumcision site once a day. Rinse area off with warm water and pat dry. Apply small amount of petroleum jelly. Fasten diaper over penis snugly enough so that it does not move and rub the tender glans.

32)A neonate weighing 5.5 lb is prescribed to receive intravenous cefotaxime (Claforan) 50 mg/kg/dose every 6 hours for treatment of sepsis. How many mg of the medication should the nurse anticipate the patient will need for 24 hours? (Calculate to the nearest whole number.)

Answer: 500 mg Explanation: First convert the patient's weight in lb to kg by dividing by 2.2, or 5.5/2.2 = 2.5 kg. Then calculate the amount of medication needed for each dose by multiplying 50 mg × 2.5 kg = 125 mg. Then multiply the amount of medication for each dose by 4 (one dose every 6 hours) or 125 mg × 4 = 500 mg. The patient will receive 500 mg of the medication in a 24-hour period.

36)At birth a newborn measured 20 inches. What length should the nurse instruct the mother to expect the baby to be at 4 months? (Calculate the anticipated length in cm and round to the nearest whole number.)

Answer: 61 cm Explanation: The conversion 2.54 cm = 1 inch will be used. If the initial length was 20 inches, convert this to cm by multiplying 20 × 2.54 = 50.8 cm. If the average growth is approximately one inch per month for the first 6 months, then multiply 2.54 × 4 = 10.16 cm and add this amount to the birth length of 50.8 cm or 10.16 cm + 50.8 cm = 60.96. With rounding, the mother can expect the infant to be 61 cm in length.

37)The nurse is instructing a new mother on the amount and frequency of bottle-feeding for her newborn. The mother plans to use formula that is available in 6-ounce cans. If the infant ingests 25 mL for each of 8 feedings per day, how many cans of formula should the mother have available for a week? (Round to the nearest whole number.)

Answer: 8 cans Explanation: One ounce is equal to 30 mL. For a 6-ounce can, the amount of formula is 30 × 6 or 180 mL. For one day, the infant will ingest 25 mL × 8 feedings or 200 mL. For 7 days the infant will ingest 200 mL × 7 = 1400 mL. To determine the number of cans of formula needed divide the weekly total of 1400 mL by 180 mL or 1400/180 = 7.78 or 8 cans of formula is needed

35)At 3 weeks a newborn weighs 8 lbs. 1 ounce. What percent of this body weight should the nurse explain to the mother as being water? (Calculate the average weight in ounces to the first decimal point.)

Answer: 93.5 ounces Explanation: Approximately 70% to 75% of the newborn's body weight is water. First determine the infant's weight in ounces by multiplying 8 × 16 ounces = 128 ounces + 1 ounce = 129 ounces. Then multiply the weight in ounces by 70% and then by 75%: 129 ounces × 70% = 90.3 ounces; 129 ounces × 75% = 96.75. Then add the values of 70% and 75% and divide by 2 to determine the average: 90.3 + 96.75 = 187.05/2 = 93.525 ounces. To determine this weight in pounds divide by 16, or 93.525/16 = 5.84 of the infant's weight is water. To determine the weight in pounds and ounces: .84/100 = x/16; 1344 = 100x; 1344/100 = 13.44 ounces. The percentage of the infant's weight that is water is 5 pounds, 13.44 ounces.

9) The nurse assesses four newborns. Which of the following assessment findings would place a newborn at risk for developing physiologic jaundice? A) Cephalohematoma B) Mongolian spots C) Telangiectatic nevi D) Molding

Answer: A Explanation: A) A cephalohematoma is a collection of blood resulting from ruptured blood vessels between the surface of a cranial bone and the periosteal membrane. They may be associated with physiologic jaundice, because there are extra red blood cells being destroyed within the cephalohematoma. B) Mongolian spots are macular areas of bluish-black pigmentation on the dorsal area of the buttocks. C) Telangiectatic nevi are pale pink or red spots found on the eyelids, nose, lower occipital bone, or nape of the neck. D) Molding is caused by overriding of the cranial bones.

3) The nurse is caring for four newborns who have recently been admitted to the newborn nursery. Which labor event puts the newborn at risk for an alteration of health? A) The infant's mother has group B streptococcal (GBS) disease. B) The infant's mother had an IV of lactated Ringer's solution. C) The infant's mother had a labor that lasted 12 hours. D) The infant's mother had a cesarean birth with her last child

Answer: A Explanation: A) A common cause of neonatal distress is early-onset group B streptococcal (GBS) disease. Infected mothers transmit GBS infection to their infants during labor and birth. All infants of mothers identified as at risk should be assessed and observed for signs and symptoms of sepsis. B) An IV of lactated Ringer's solution will not affect the newborn's blood sugar. C) A 12-hour labor is normal. D) Having had a cesarean with her last child poses risk factors for the mother during labor, but does not affect this newborn.

25)The student nurse notices that a newborn weighs less today compared with the newborn's birth weight three days ago. The nursing instructor explains that newborns lose weight following birth due to which of the following? A) A shift of intracellular water to extracellular spaces. B) Loss of meconium stool. C) A shift of extracellular water to intracellular spaces. D) The sleep-wake cycle.

Answer: A Explanation: A) A shift of intracellular water to extracellular space and insensible water loss account for the 5% to 10% weight loss. B) Loss of meconium stool does not effect this amount of weight loss. C) A shift of intracellular water to extracellular space and insensible water loss account for the 5% to 10% weight loss. D) The sleep-wake cycle does not effect this amount of weight loss

23)What condition is due to poor peripheral circulation? A) Acrocyanosis B) Mottling C) Harlequin sign D) Jaundice

Answer: A Explanation: A) Acrocyanosis is a bluish discoloration of the hands and feet that may be present in the first 24 hours after birth and is due to poor peripheral circulation, which results in vasomotor instability and capillary stasis, especially when the baby is exposed to cold. B) Mottling is a lacy pattern of dilated blood vessels under the skin and occurs as a result of general circulation fluctuations. C) Harlequin sign (clown) color change is a deep color that develops over one side of the newborn's body while the other side remains pale, so that the skin resembles a clown's suit. D) Jaundice is a yellowish discoloration of skin and mucous membranes first detectable on the face and the mucous membranes of the mouth.

19)The nurse is teaching the parents of an infant with an inborn error of metabolism how to care for the infant at home. What information does teaching include? A) Specially prepared formulas B) Cataract problems C) Low glucose concentrations D) Administration of thyroid medication

Answer: A Explanation: A) An afflicted PKU infant can be treated by a special diet that limits ingestion of phenylalanine. Special formulas low in phenylalanine, such as Phenyl-Free 1 and Phenex-1, are available. B) Cataracts are associated with infants who have galactosemia. C) Low glucose concentrations are not an indication an inborn error of metabolism. D) Thyroid medication is given to infants with congenital hypothyroidism.

1) A nursing instructor is demonstrating an assessment on a newborn using the Ballard gestational assessment tool. The nurse explains that which of the following tests should be performed after the first hour of birth, when the newborn has had time to recover from the stress of birth? A) Arm recoil B) Square window sign C) Scarf sign D) Popliteal angle

Answer: A Explanation: A) Arm recoil is slower in healthy but fatigued newborns after birth; therefore, arm recoil is best elicited after the first hour of birth, when the baby has had time to recover from the stress of birth. B) The square window sign does not have to be assessed after the first hour of birth. C) The scarf sign does not have to be assessed after the first hour of birth. D) The popliteal angle does not have to be assessed after the first hour of birth

6) The pediatric clinic nurse is reviewing lab results with a 2-month-old infant's mother. The infant's hemoglobin has decreased since birth. Which statement by the mother indicates the need for additional teaching? A) "My baby isn't getting enough iron from my breast milk." B) "Babies undergo physiologic anemia of infancy." C) "This results from dilution because of the increased plasma volume." D) "Delaying the cord clamping did not cause this to happen."

Answer: A Explanation: A) At 2 months of age, infants increase their plasma volume, which results in physiologic anemia. This condition is not related to iron in the breast milk. B) This initial decline in hemoglobin creates a phenomenon known as physiologic anemia of the newborn. C) Hemoglobin values fall, mainly from a decrease in red cell mass rather than from the dilutional effect of increasing plasma volume. D) Early or delayed cord clamping does not affect hemoglobin levels at this age.

12)The nurse wishes to demonstrate to a new family their infant's individuality. Which assessment tool would be most appropriate for the nurse to use? A) Brazelton Neonatal Behavioral Assessment Scale B) New Ballard Score C) Dubowitz gestational age scale D) Ortolani maneuver

Answer: A Explanation: A) Brazelton Neonatal Behavioral Assessment Scale is an assessment tool that identifies the newborn's repertoire of behavioral responses to the environment and documents the newborn's neurologic adequacy and capabilities. B) Ballard developed the estimation of gestational age by maturity rating. C) The Dubowitz assessment tool assesses physical characteristics and neurological or neuromuscular development. D) The Ortolani maneuver is an assessment technique to evaluate for hip dislocation or hip instability.

18)Which of the following is a localized, easily identifiable soft area of the infant's scalp, generally resulting from a long and difficult labor or vacuum extraction? A) Caput succedaneum B) Cephalohematoma C) Molding D) Depressed fontanelles

Answer: A Explanation: A) Caput succedaneum is a localized, easily identifiable soft area of the scalp, generally resulting from a long and difficult labor or vacuum extraction. B) Cephalohematoma is a collection of blood resulting from ruptured blood vessels between the surface of a cranial bone and the periosteal membrane. C) The head may appear asymmetric in the newborn who had a vertex presentation. This asymmetry (molding) is caused by the overriding of the cranial bones during labor and birth. D) A depressed fontanelle indicates dehydration.

13)The nurse assesses the gestational age of a newborn and informs the parents that the newborn is premature. Which of the following assessment findings is not congruent with prematurity? A) Cry is weak and feeble B) Clitoris and labia minora are prominent C) Strong sucking reflex D) Lanugo is plentiful

Answer: C Explanation: A) Findings that indicate prematurity include a weak cry. B) Findings that indicate prematurity include a prominent clitoris and labia minora. C) Poor suck, gag, and swallow reflexes are characteristic of a preterm newborn. D) Findings that indicate prematurity include lanugo that is plentiful and widely distributed.

22.The postpartum client who delivered 2 days ago has developed endometritis. Which entry would the nurse expect to find in this client's chart? A) "Cesarean birth after extended labor with ruptured membranes." B) "Unassisted childbirth and afterbirth." C) "External fetal monitoring used throughout labor." D) "The client has history of pregnancy-induced hypertension."

Answer: A Explanation: A) Cesarean birth is the single most significant risk factor for postpartum endometritis, along with prolonged premature rupture of the amniotic membranes (PPROM). B) Instrument-assisted childbirth (vacuum or forceps) and manual removal of the placenta are risk factors for postpartum endometritis. C) Use of fetal scalp electrode or intrauterine pressure catheter for internal monitoring during labor are risk factors for postpartum endometritis. D) Pregnancy-induced hypertension is not a risk factor for postpartum endometritis

7) Which of the following is a benefit of delayed umbilical cord clamping for the preterm infant? A) Fewer infants require blood transfusion for anemia B) Fewer infants require blood transfusion for high blood pressure C) Increase in the incidence of intraventricular hemorrhage D) Increase in incidence of infant breastfeeding

Answer: A Explanation: A) Clinical trials in preterm infants found that delaying umbilical cord clamping was associated with fewer infants who required blood transfusion for anemia. B) Clinical trials in preterm infants found that delaying umbilical cord clamping was associated with fewer infants who required blood transfusion for low blood pressure. C) Delayed umbilical cord clamping shows a significant reduction in the incidence of intraventricular hemorrhage. D) Delayed umbilical cord clamping does not impact the incidence of breastfeeding.

28)Which nonspecific immune mechanism helps the ability of antibodies and phagocytic cells to clear pathogens from an organism? A) Complement B) Coagulation C) Inflammatory response D) Phagocytosis

Answer: A Explanation: A) Complement helps or "complements" the ability of antibodies and phagocytic cells to clear pathogens from an organism. B) Coagulation is the process by which blood forms a clot. C) Inflammatory response is the complex biologic response of vascular tissues to harmful stimuli such as pathogens, damaged cells or irritants. D) Phagocytosis is a major mechanism to remove pathogens and cell debris

12)A newborn delivered at term is being discharged. The parents ask the nurse how to keep their baby warm. The nurse knows additional teaching is necessary if a parent states which of the following? A) "A quick cool bath will help wake up my son for feedings." B) "I can check my son's temperature under his arm." C) "My baby should be dressed warmly, with a hat." D) "Cuddling my son will help to keep him warm."

Answer: A Explanation: A) Cool baths will chill a newborn, and should not be given. Bathing under warm water is ideal. B) The axilla is the preferred site for checking a newborn's temperature. C) Adequate clothing is needed to keep an infant warm. A snug cap placed on the infant's head reduces heat loss further. D) Encourage the mother to snuggle with the newborn under blankets to keep him or her warm.

2) Before drying off the newborn after birth, which assessment finding should the nurse document to ensure an accurate gestational rating on the Ballard gestational assessment tool? A) Amount and area of vernix coverage B) Creases on the sole C) Size of the areola D) Body surface temperature

Answer: A Explanation: A) Drying the baby after birth will disturb the vernix and potentially alter the gestational age criterion. The nurse should document the amount and areas of vernix coverage before drying the newborn. B) Creases on the sole are not affected by drying the newborn. C) The size of the areola is not affected by drying the newborn. D) Body surface temperature is not part of the Ballard gestational assessment tool.

4) A postpartum client has inflamed hemorrhoids. Which nursing intervention would be appropriate? A) Encourage sitz baths. B) Position the client in the supine position. C) Avoid stool softeners. D) Decrease fluid intake.

Answer: A Explanation: A) Encouraging sitz baths is the correct approach because moist heat decreases inflammation and provides for comfort. B) Position the client in a side-lying position. C) Avoiding stool softeners would put the client at risk for constipation and increase the likelihood of inflammation. D) Decreasing fluid intake would put the client at risk for constipation and would be contraindicated for reducing inflammation.

11)A 3-month-old baby who was born at 25 weeks has been exposed to prolonged oxygen therapy. The nurse explains to the parents that due to oxygen therapy, their infant is at a greater risk for which of the following? A) Visual impairment B) Hyperthermia C) Central cyanosis D) Sensitive gag reflex

Answer: A Explanation: A) Extremely premature newborns are particularly susceptible to injury of the delicate capillaries of the retina, causing characteristic retinal changes known as retinopathy of prematurity (ROP). Judicious use of supplemental oxygen therapy in the premature infant has become the norm. B) Hypothermia is more common in premature infants. C) Central cyanosis can be caused by decreased oxygen. D) An absent or decreased gag reflex is more common in premature infants.

16.The postpartum multipara is breastfeeding her new baby. The client states that she developed mastitis with her first child, and asks whether there is something she can do to prevent mastitis this time. What would the best response of the nurse be? A) "Massage your breasts on a daily basis, and if you find a hardened area, massage it towards the nipple." B) "Most first-time moms experience mastitis. It is really quite unusual for a woman having her second baby to get it again." C) "Apply cabbage leaves to any areas that feel thickened or firm to relieve the swelling." D) "Take your temperature once a day. This will help you to pick up the infection early, before it becomes severe."

Answer: A Explanation: A) If the mother finds that one area of her breast feels distended or lumpy, she can massage the lumpy area toward the nipple as the infant nurses. B) This statement is not accurate. Most first-time moms do not experience mastitis. C) Cabbage leaves are applied to suppress lactation, not prevent mastitis. D) The onset of mastitis is quite rapid, and taking the temperature daily is not likely to be helpful for catching early onset of the infection.

8) Which assessment findings by the nurse would require obtaining a blood glucose level on the newborn? A) Jitteriness B) Sucking on fingers C) Lusty cry D) Axillary temperature of 98°

Answer: A Explanation: A) Jitteriness of the newborn is associated with hypoglycemia. Aggressive treatment is recommended after a single low blood glucose value if the infant shows this symptom. B) Sucking on the fingers is a normal finding. C) A lusty cry is a normal finding. D) An axillary temperature of 98°F is a normal finding.

19)The nurse is evaluating the effectiveness of phototherapy on a newborn. Which evaluation indicates a therapeutic response to phototherapy? A) The newborn maintains a normal temperature B) An increase ofserum bilirubin levels C) Weight loss D) Skin blanching yellow

Answer: A Explanation: A) Maintenance of temperature is an important aspect of phototherapy because the newborn is naked except for a diaper during phototherapy.The isolette helps the infant maintain his or her temperature while undressed. B) Phototherapy is a primary intervention that is used for the prevention of hyperbilirubinemia, to halt bilirubin levels from climbing dangerously high. C) Weight loss is a sign of developing dehydration in the newborn. The newborn should be weighed daily. D) Yellowing in the skin should disappear with effective phototherapy.

3) The nurse would expect a physician to prescribe which medication to a postpartum client with heavy bleeding and a boggy uterus? A) Methylergonovine maleate (Methergine) B) Rh immune globulin (RhoGAM) C) Terbutaline (Brethine) D) Docusate (Colace)

Answer: A Explanation: A) Methylergonovine maleate is the drug used for the prevention and control of postpartum hemorrhage. B) RhoGAM is a medicine given by intramuscular injection that is used to prevent the immunological condition known as Rh disease (or hemolytic disease of the newborn). C) Terbutaline (Brethine) is a β2-adrenergic receptor agonist used as a "reliever" inhaler in the management of asthma. D) Docusate is a stool softener.

7) The nurse is caring for the newborn of a diabetic mother. Which of the following should be included in the nurse's plan of care for this newborn? A) Offer early feedings. B) Administer an intravenous infusion of glucose. C) Assess for hypercalcemia. D) Assess for hyperbilirubinemia immediately after birth.

Answer: A Explanation: A) Newborns of diabetic mothers may benefit from early feedings, as they are extremely valuable in maintaining normal metabolism and lowering the possibility of such complications as hypoglycemia and hyperbilirubinemia. B) If normal glucose levels cannot be maintained with oral feeding, an intravenous (IV) infusion of glucose will be necessary. C) The newborn should be assessed for hypocalcemia. D) Hyperbilirubinemia can occur 48 to 72 hours after birth.

25)In planning care for the fetal alcohol syndrome (FAS) newborn, which intervention would the nurse include? A) Allow extra time with feedings. B) Assign different personnel to the newborn each day. C) Place the newborn in a well-lit room. D) Monitor for hyperthermia.

Answer: A Explanation: A) Newborns with fetal alcohol syndrome have feeding problems. Because of their feeding problems, these infants require extra time and patience during feedings. B) It is important to provide consistency in the staff working with the baby and parents and to keep personnel and visitors to a minimum at any one time. C) The FASD baby is most comfortable in a quiet, minimally stimulating environment. D) Nursing care of the FASD newborn is aimed at avoiding heat loss.

10)The mother of a premature newborn questions why a gavage feeding catheter is placed in the mouth of the newborn and not in the nose. What is the nurse's best response? A) "Most newborns are nose breathers." B) "The tube will elicit the sucking reflex." C) "A smaller catheter is preferred for feedings." D) "Most newborns are mouth breathers."

Answer: A Explanation: A) Orogastric insertion is preferable to nasogastric because most infants are obligatory nose breathers. B) The tube or gavage feeding method is used with preterm infants who lack or have a poorly coordinated suck-swallow-breathing pattern. C) A small catheter is used for a nasogastric tube to minimize airway obstruction. D) Orogastric insertion is preferable to nasogastric because most infants are obligatory nose breathers.

26)A postpartum client calls the nursery to report that her newborn's umbilical cord stump is draining, and has a foul odor. What is the nurse's best response? A) "Take your newborn to the pediatrician." B) "Cover the cord stump with gauze." C) "Apply Betadine around the cord stump." D) "This is normal during healing."

Answer: A Explanation: A) Parents should check each day for any odor, oozing of greenish yellow material, or reddened areas around the cord. They should report to the healthcare provider any signs of infection. B) Parents should fold diapers below the umbilical cord to air-dry the cord. Contact with wet or soiled diapers slows the drying process and increases the possibility of infection. C) Betadine is not used on the cord stump. D) These symptoms are not normal.

18)The home care nurse is examining a 3-day-old infant. The child's skin on the sternum is yellow when blanched with a finger. The parents ask the nurse why jaundice occurs. What is the best response from the nurse? A) "The liver of an infant is not fully mature, and doesn't conjugate the bilirubin for excretion." B) "The infant received too many red blood cells after delivery because the cord was not clamped immediately." C) "The yellow color of your baby's skin indicates that you are breastfeeding too often." D) "This is an abnormal finding related to your baby's bowels not excreting bilirubin as they should."

Answer: A Explanation: A) Physiologic jaundice is a common occurrence, and peaks at 3 to 5 days in term infants. The reduction in hepatic activity, along with a relatively large bilirubin load, decreases the liver's ability to conjugate bilirubin and increases susceptibility to jaundice. B) The conjugation of bilirubin has nothing to do with cord clamping. C) Prevention and treatment of early breastfeeding jaundice includes encouraging frequent (every 2 to 3 hours) breastfeeding. D) Direct bilirubin is excreted into the bile ducts and duodenum. The conjugated bilirubin then progresses down the intestines, where bacteria transform it into urobilinogen and stercobilinogen. Stercobilinogen is not reabsorbed, but is excreted as a yellow-brown pigment in the stools.

14)The mother of a 3-day-old infant calls the clinic and reports that her baby's skin is turning slightly yellow. What should the nurse explain to the mother? A) Physiologic jaundice is normal, and peaks at this age. B) The newborn's liver is not working as well as it should. C) The baby is yellow because the bowels are not excreting bilirubin. D) The yellow color indicates that brain damage might be occurring.

Answer: A Explanation: A) Physiologic jaundice occurs soon after birth. Bilirubin levels peak at 3 to 5 days in term infants. B) The liver of an infant is not fully mature at this point. C) The liver of an infant conjugates the bilirubin, which is then excreted through the bowels. D) Unmonitored and untreated severe hyperbilirubinemia may progress to excessive levels that are associated with bilirubin neurotoxicity. An infant with severe jaundice would have a high level of yellow skin color, but this infant is only slightly yellow.

15)The community nurse is working with a client from Southeast Asia who has delivered her first child. Her mother has come to live with the family for several months. The nurse understands that the main role of the grandmother while visiting is to do which of the following? A) Help the new mother by allowing her to focus on resting and caring for the baby. B) Teach her son-in-law the right way to be a father because this is his first child. C) Make sure that her daughter does not become abusive towards the infant. D) Pass on cultural values and beliefs to the newborn grandchild.

Answer: A Explanation: A) Rest, seclusion, and dietary restraint practices in many traditional non-Western cultures (South Asian groups) are designed to assist the woman and her baby during postpartum vulnerable periods. B) The new father might be taught some skills either directly or indirectly through observation, but this is not the most important role of the grandmother. C) The new grandmother does not assume that her daughter will be abusive toward the infant. D) An infant is too young to be taught values and beliefs

25)Which of the following conditions would predispose a client for thrombophlebitis? A) Severe anemia B) Cesarean delivery C) Anorexia D) Hypocoagulability

Answer: A Explanation: A) Severe anemia would predispose a client for thrombophlebitis. B) A traumatic delivery would predispose a client for thrombophlebitis. C) Obesity, not anorexia, would predispose a client for thrombophlebitis. D) Hypercoagulability would predispose a client for thrombophlebitis.

24)The parents of a newborn are receiving discharge teaching. The nurse explains that the infant should have several wet diapers per day. Which statement by the parents indicates that further education is necessary? A) "Our baby was born with kidneys that are too small." B) "A baby's kidneys don't concentrate urine well for several months." C) "Feeding our baby frequently will help the kidneys function." D) "Kidney function in an infant is very different from that in an adult

Answer: A Explanation: A) Size of the kidneys is rarely an issue. B) The ability to concentrate urine fully is attained by 3 months of age. C) Feeding practices may affect the osmolarity of the urine but have limited effect on concentration of the urine. D) The neonate's ability to dilute urine is fully developed, but concentrating ability is limited.

22)The nurse is instructing parents of a newborn about voiding and stool characteristics. Which of the following would be considered an abnormal pattern? A) Large amounts of uric acid crystals in the first days of life B) At least 6 to 10 wet diapers a day after the first few days of life C) 1 to 2 stools a day for a formula-fed baby D) Urine that is straw to amber color without foul smell

Answer: A Explanation: A) Small, not large, amounts of uric acid crystals are normal in the first days of life. B) 6 to 10 wet diapers a day after the first few days of life is normal. C) 1 to 2 stools a day for a formula-fed baby is normal. D) Urine that is straw to amber color without foul smell is normal.

31)The parents of a newborn have just been told their infant has tetralogy of Fallot. The parents do not seem to understand the explanation given by the physician. What statement by the nurse is best? A) "With this defect, not enough of the blood circulates through the lungs, leading to a lack of oxygen in the baby's body." B) "The baby's aorta has a narrowing in a section near the heart that makes the left side of the heart work harder." C) "The blood vessels that attach to the ventricles of the heart are positioned on the wrong sides of the heart." D) "Your baby's heart doesn't circulate blood well because the left ventricle is smaller and thinner than normal."

Answer: A Explanation: A) Tetralogy of Fallot is a cyanotic heart defect that comprises four abnormalities: pulmonary stenosis, ventricular septal defect, overriding aorta, and right ventricle hypertrophy. The severity of symptoms depends on the degree of pulmonary stenosis, the size of the ventricular septal defect, and the degree to which the aorta overrides the septal defect. B) This describes coarctation of the aorta and is characterized by a narrowed aortic lumen. The lesion produces an obstruction to the flow of blood through the aorta, causing an increased left ventricular pressure and workload, minimizing systemic circulation of blood. C) This describes complete transposition of great vessels and is an embryologic defect caused by a straight division of the bulbar trunk without normal spiraling. As a result, the aorta originates from the right ventricle, and the pulmonary artery from the left ventricle resulting in a parallel circulatory system. An abnormal communication between the two circulations must be present to sustain life. D) This describes hypoplastic left heart syndrome which is the underdevelopment of the left side of the heart including aortic valve atresia, severe mitral valve stenosis, and small left ventricle.

6) The nurse is making an initial assessment of the newborn. Which of the following data would be considered normal? A) Chest circumference 31.5 cm, head circumference 33.5 cm B) Chest circumference 30 cm, head circumference 29 cm C) Chest circumference 38 cm, head circumference 31.5 cm D) Chest circumference 32.5 cm, head circumference 36 cm

Answer: A Explanation: A) The average circumference of the head at birth is 32 to 37 cm. Average chest circumference ranges from 30 to 35 cm at birth. The circumference of the head is approximately 2 cm greater than the circumference of the chest at birth. Answer 1 is the only choice in which both the chest and head circumferences fall within the norm in terms of actual size and comparable size. B) The average circumference of the head at birth is 32 to 37 cm. Average chest circumference ranges from 30 to 35 cm at birth. The circumference of the head is approximately 2 cm greater than the circumference of the chest at birth. Answer 1 is the only choice in which both the chest and head circumferences fall within the norm in terms of actual size and comparable size. C) The average circumference of the head at birth is 32 to 37 cm. Average chest circumference ranges from 30 to 35 cm at birth. The circumference of the head is approximately 2 cm greater than the circumference of the chest at birth. Answer 1 is the only choice in which both the chest and head circumferences fall within the norm in terms of actual size and comparable size. D) The average circumference of the head at birth is 32 to 37 cm. Average chest circumference ranges from 30 to 35 cm at birth. The circumference of the head is approximately 2 cm greater than the circumference of the chest at birth. Answer 1 is the only choice in which both the chest and head circumferences fall within the norm in terms of actual size and comparable size.

17)A newborn is receiving phototherapy. Which intervention by the nurse would be most important? A) Measurement of head circumference B) Encouraging the mother to stop breastfeeding C) Stool blood testing D) Assessment of hydration status

Answer: D Explanation: A) Phototherapy does not affect head circumference. B) Breastfeeding most likely can be continued. C) The stools do not need to be tested for blood. D) Infants undergoing phototherapy treatment have increased water loss and loose stools as a result of bilirubin excretion. This increases their risk of dehydration.

28.The client delivered by cesarean birth 3 days ago and is being discharged. Which statement should the nurse include in the discharge teaching? A) "If your incision becomes increasingly painful, call the doctor." B) "It is normal for the incision to ooze greenish discharge in a few days." C) "Increasing redness around the incision is a part of the healing process." D) "A fever is to be expected because you had a surgical delivery

Answer: A Explanation: A) The client should call the doctor if the incision becomes increasingly painful. After cesarean delivery, wound infection is most often associated with concurrent endometritis. The wound is typically red, indurated, tender at the margins, and draining purulent exudate. Some women have cellulitis without actual purulent drainage. B) Green drainage is not an acceptable symptom. C) The client should call the doctor if the incision becomes increasingly painful. Cesarean wound infections are characterized by increasing redness and tenderness at the margins. D) A fever could be a symptom of infection and this client should call the doctor

27)The mother of a 2-day-old male has been informed that her child has sepsis. The mother is distraught and says, "I should have known that something was wrong. Why didn't I see that he was so sick?" What is the nurse's best reply? A) "Newborns have immature immune function at birth, and illness is very hard to detect." B) "Your mothering skills will improve with time. You should take the newborn class." C) "Your baby didn't get enough active acquired immunity from you during the pregnancy." D) "The immunity your baby gets in utero doesn't start to function until he is 4 to 8 weeks old."

Answer: A Explanation: A) The immune responses in neonates are usually functionally impaired when compared with adults. B) This response does not address the physiology of neonatal infection, and is not therapeutic because it is blaming. C) The pregnant woman forms antibodies in response to illness or immunization called active acquired immunity. Neonatal defense against infections in utero or after delivery is dependent on maternal immunity. D) When antibodies are transferred to the fetus in utero, passive acquired immunity results because the fetus does not produce the antibodies itself.

23)Which of the following behaviors noted in the postpartum client would require the nurse to assess further? A) Responds hesitantly to infant cries. B) Expresses satisfaction about the sex of the baby. C) Friends and family visit the client and give advice. D) Talks to and cuddles with the infant frequently.

Answer: A Explanation: A) The mother tends to respond verbally to any sounds emitted by the newborn, such as cries, coughs, sneezes, and grunts. Responding hesitantly to infant cries suggests further assessment is needed to determine what the mother is feeling. B) Expressing satisfaction about the sex of the baby is usually a positive sign. C) Birth centers are especially geared toward the inclusion of the family in the birth process. In the hospital setting, the advent of open visiting hours and rooming-in permits siblings and grandparents to participate in the attachment process. D) Talking to and cuddling with the infant frequently is a behavior that facilitates attachment.

32)The nurse is teaching a group of new parents about their infants. The infants are all 4 weeks of age or younger. Which statement should the nurse include? A) "Your baby will respond to you the most if you look directly into his eyes and talk to him." B) "Each baby is different. Don't try to compare your infant's behavior with any other child's behavior." C) "If the sound level around your baby is high, the baby will wake up and be fussy or cry." D) "If your baby is a cuddler, it is because you rocked and talked to her during your pregnancy."

Answer: A Explanation: A) The parents' visual (en face) and auditory (soft, continuous voice) presence stimulates their infant to orient to them. B) Although each infant is unique, there are certain predictable norms to observe for when assessing for neurological normalcy or impairment. C) Some infants become overstimulated when excessive noise is present, but more habituate to the sound and sleep. D) Cuddling is a social behavior that correlates with personality, but it has not been linked to any prenatal activities.

9) A nursing instructor is demonstrating how to perform a heel stick on a newborn. To obtain an accurate capillary hematocrit reading, what does the nursing instructor tell the student do? A) Rub the heel vigorously with an isopropyl alcohol swab prior to obtaining blood. B) Use a previous puncture site. C) Cool the heel prior to obtaining blood. D) Use a sterile needle and aspirate.

Answer: A Explanation: A) The site should be cleaned by rubbing vigorously with a 70% isopropyl alcohol swab. The friction produces local heat, which aids vasodilation. B) A microlancet is used to make the puncture in an unpunctured site. C) The heel should not be cooled. D) A microlancet, not a needle, is used to make the puncture.

9) The pregnant client at 41 weeks is scheduled for labor induction. She asks the nurse whether induction is really necessary. What response by the nurse is best? A) "Babies can develop postmaturity syndrome, which refers to a number of complications that can occur after 42 weeks of pregnancy." B) "When infants are born 2 or more weeks after their due date, they have meconium in the amniotic fluid." C) "Sometimes the placenta ages excessively, and we want to take care of that problem before it happens." D) "The doctor wants to be proactive in preventing any problems with your baby if he gets any bigger."

Answer: A Explanation: A) The term postmaturity applies to the infant who is born after 42 completed weeks of gestation and demonstrates characteristics of postmaturity syndrome. B) Although this statement is partially true, meconium-stained amniotic fluid is not always present or the only complication of postmaturity syndrome. C) Although this statement is true, it is too vague. It is better to be specific and call postmaturity syndrome by its name. D) Although this is true, the answer is incomplete. The risk of postmaturity syndrome is also an issue.

18)The nurse is observing a student nurse who is caring for a neonate undergoing intensive phototherapy. Which action by the student nurse indicates an understanding of how to provide this care? A) Urine specific gravity is assessed at each voiding. B) Eye coverings are left off to help keep the baby calm. C) Temperature is checked every 6 hours. D) The infant is taken out of the isolette for diaper changes.

Answer: A Explanation: A) This action is correct. Specific gravity provides one measure of urine concentration. Highly concentrated urine is associated with a dehydrated state. Weight loss is also a sign of developing dehydration in the newborn. B) Eyes should be covered at all times. C) Six hours is too long. Vital signs should be monitored every 4 hours with axillary temperatures. D) The isolette helps the infant maintain his or her temperature while undressed. The diaper should be changed while the infant is under the lights in the isolette, as care activities should be clustered.

2) The nurse is caring for several pregnant clients. Which client should the nurse anticipate is most likely to have a newborn at risk for mortality or morbidity? A) 37-year-old, with a history of multiple births and preterm deliveries who works in a chemical factory B) 23-year-old of low socioeconomic status, unmarried C) 16-year-old who began prenatal care at 30 weeks D) 28-year-old with a history of gestational diabetes

Answer: A Explanation: A) This client is at greatest risk because she has multiple risk factors: age over 35, high parity, history of preterm birth, and exposure to chemicals that might be toxic. B) The main risk factor for this client is her low socioeconomic status. C) This client has two risk factors: young age and late onset of prenatal care. D) This client's only risk factor is the history of gestational diabetes.

25.The client delivered vaginally 2 hours ago after receiving an epidural analgesia. She has a slight tingling sensation in both lower extremities, but normal movement. She sustained a second-degree perineal laceration. Her perineum is edematous and ecchymotic. What should the nurse include in the plan of care for this client? A) Assist the client to the bathroom in 2 hours to void. B) Place a Foley catheter now. C) Apply warm packs to the perineum three times a day. D) Allow the client to rest for the next 8 hours.

Answer: A Explanation: A) This client is at risk for urinary retention and bladder overdistention. Overdistention occurs postpartum when the woman is unable to empty her bladder, usually because of trauma or the effects of anesthesia. After the effects of anesthesia have worn off, if the woman cannot void, postpartum urinary retention is highly indicative of a urinary tract infection (UTI). Assisting the client to the bathroom is the most likely intervention that will prevent urinary retention. B) A Foley catheter is not indicated at this time. C) Cold packs will help decrease the perineal edema; warm packs would increase the edema. D) Waiting 8 hours to reassess the bladder is too long.

17)The mother of a 16-week-old infant is concerned because she cannot feel the posterior fontanelle on her infant. Which response by the nurse would be most appropriate? A) "It is normal for the posterior fontanelle to close by 8 to 12 weeks after birth." B) "Bring your infant to the clinic immediately." C) "This is due to overriding of the cranial bones during labor." D) "Your baby must be dehydrated."

Answer: A Explanation: A) This is a normal finding at 16 weeks. The posterior fontanelle closes within 8 to 12 weeks. B) There is no reason to bring the infant to the clinic. C) Overriding of the cranial bones is referred to as molding, and diminishes within a few days following birth. D) Fontanelles can be depressed when the infant is dehydrated.

33)The nurse is answering phone calls at the pediatric clinic. Which call should the nurse return first? A) Mother of a 2-week-old infant who doesn't make eye contact when talked to B) Father of a 1-week-old infant who sleeps through the noise of an older sibling C) Father of a 6-day-old infant who responds more to mother's voice than to father's voice D) Mother of a 3-week-old infant who has begun to suck on the fingers of the right hand

Answer: A Explanation: A) This is an abnormal finding. Orientation to the environment is determined by an ability to respond to cues given by others and by a natural ability to fix on and to follow a visual object horizontally and vertically. Inability or lack of response may indicate visual or auditory problems. B) Sleeping though noise is habituation, and is an expected behavior. C) The newborn can discriminate the individual characteristics of the human voice and is especially sensitive to sound levels within the normal conversation range. D) Self-consolatory behaviors such as sucking on fists, thumbs, or fingers are normal findings.

20)At birth, an infant weighed 6 pounds 12 ounces. Three days later, he weighs 5 pounds 2 ounces. What conclusion should the nurse draw regarding this newborn's weight? A) This weight loss is excessive. B) This weight loss is within normal limits. C) This weight gain is excessive. D) This weight gain is within normal limits

Answer: A Explanation: A) This newborn has lost more than 10% of the birth weight; this weight loss is excessive. Following birth, caloric intake is often insufficient for weight gain until the newborn is 5 to 10 days old. During this time there may be a weight loss of 5% to 10% in term newborns. B) This weight loss is greater than the expected 5% to 10%. C) This is not a weight gain. D) This is not a weight gain.

10)The nurse is preparing new parents for discharge with their newborn. The father asks the nurse why the baby's head is so pointed and puffy-looking. What is the best response by the nurse? A) "His head is molded from fitting through the birth canal. It will become more round." B) "We refer to that as 'cone head,' which is a temporary condition that goes away." C) "It might mean that your baby sustained brain damage during birth, and could have delays." D) "I think he looks just like you. Your head is much the same shape as your baby's."

Answer: A Explanation: A) This statement is accurate and directly answers the father's question. B) Although nursing staff might refer to a molded head as "cone head" and the shape is temporary, it is better to be more specific in explaining why the head is shaped as it is. In addition, this answer does not answer the "why" question posed by the father. C) A molded head shape does not indicate brain damage. Molding is normal and transient. D) Although this might be true, it is better to give a factual answer that does not imply that the father's head is abnormally shaped. This answer could be perceived as insulting.

5) The nurse is caring for a 2-hour-old newborn whose mother is diabetic. The nurse assesses that the newborn is experiencing tremors. Which nursing action has the highest priority? A) Obtain a blood calcium level. B) Take the newborn's temperature. C) Obtain a bilirubin level. D) Place a pulse oximeter on the newborn.

Answer: A Explanation: A) Tremors are a sign of hypocalcemia. Diabetic mothers tend to have decreased serum magnesium levels at term. This could cause secondary hypoparathyroidism in the infant. B) Body temperature might be necessary to monitor, but obtaining a blood calcium level takes priority for this newborn. C) Bilirubin level might be necessary to monitor, but obtaining a blood calcium level takes priority for this newborn. D) Oxygen saturation might be necessary to monitor, but obtaining a blood calcium level takes priority for this newborn.

20)Before the newborn and mother are discharged from the birthing unit, the nurse teaches the parents about newborn screening tests that includes which of the following? A) Preeclampsia screening B) Congenital kidney disease screening C) Visual screening D) Hearing screening

Answer: D Explanation: A) Preeclampsia is a maternal condition, and not part of the newborn screening tests. B) Congenital heart disease screening, not kidney disease screening, is part of the newborn screening tests. C) Visual screening is not part of newborn screening tests. D) Newborn screening tests include hearing screening tests.

4. The client has experienced a hemorrhage at 6 hours postpartum. After controlling the hemorrhage, the client's partner asks what would cause a hemorrhage. How should the nurse respond? A) "Sometimes the uterus relaxes and excessive bleeding occurs." B) "The blood collected in the vagina and poured out when your partner stood up." C) "Bottle-feeding prevents the uterus from getting enough stimulation to contract." D) "The placenta had embedded in the uterine tissue abnormally."

Answer: A Explanation: A) Uterine atony (relaxation of the uterus) is the leading cause of early postpartum hemorrhage, accounting for over 50% of postpartum hemorrhage cases. B) Although blood can pool in the vagina and thus pour out when the client stands, this does not constitute a hemorrhage. C) Although breastfeeding stimulates the release of oxytocin, which causes the uterus to contract, bottle-feeding does not cause hemorrhage. D) Had the placenta embedded abnormally (as in placenta accreta), the hemorrhage would have occurred immediately after the placenta delivered.

11)The student nurse attempts to take a newborn's vital signs, but the newborn is crying. What nursing action would be appropriate? A) Place a gloved finger in the newborn's mouth. B) Take the vital signs. C) Wait until the newborn stops crying. D) Place a hot water bottle in the isolette.

Answer: A Explanation: A) Vital sign assessments are most accurate if the newborn is at rest, so measure pulse and respirations first if the baby is quiet. To soothe a crying baby, the nurse should place a moistened, unpowdered, gloved finger in the baby's mouth, and then complete the assessment while the baby suckles. B) Crying increases heart rate and respiratory rate, so vitals should not be taken when the newborn is crying. C) Assessment of vitals needs to be done at regularly timed intervals, so waiting until the newborn stops crying might cause too long a delay. D) A hot water bottle should not be placed next to the newborn because of the risk for burns.

3) During newborn resuscitation, how does the nurse evaluate the effectiveness of bag- and-mask ventilations? A) The rise and fall of the chest B) Sudden wakefulness C) Urinary output D) Adequate thermoregulation

Answer: A Explanation: A) With proper resuscitation, chest movement is observed for proper ventilation. Pressure should be adequate to move the chest wall. B) Sudden wakefulness is not associated with effectiveness of bag-and-mask ventilations. C) Urinary output is not associated with effectiveness of bag-and-mask ventilations. D) Adequate thermoregulation is not associated with effectiveness of bag-and-mask ventilations.

30)The nurse is cross-training maternal-child health unit nurses to provide home-based care for parents after discharge. Which statements indicate that additional teaching is required? Note: Credit will be given only if all correct choices and no incorrect choices are selected. Select all that apply. A) "The behavioral assessment should be done as soon after birth as possible." B) "The behavioral assessment can be performed without input from parents." C) "The behavioral assessment might be incomplete in a 1-hour home visit." D) "The behavioral assessment includes orientation and motor activity." E) "The behavioral assessment can detect neurological impairments."

Answer: A, B Explanation: A) Because the first few days after birth are a period of behavioral disorganization, the complete assessment should be done on the third day after birth. B) Parental input is required. It provides a way for the healthcare provider, in conjunction with the parents (primary caregivers), to identify and understand the individual newborn's states, temperament, capabilities, and individual behavior patterns. C) A full behavioral assessment includes the nurse observing the newborn's sleep- wake patterns, which is not likely to take place in a 1-hour home visit. D) Orientation to visual and auditory clues and motor activity are portions of the behavioral assessment. E) The behavioral assessment can detect neurological impairments.

21.A nurse suspects that a postpartum client has mastitis. Which data support this assessment? Note: Credit will be given only if all correct choices and no incorrect choices are selected. Select all that apply. A) Shooting pain between breastfeedings B) Late onset of nipple pain C) Pink, flaking, pruritic skin of the affected nipple D) Nipple soreness when the infant latches on E) Pain radiating to the underarm area from the breast

Answer: A, B, C Explanation: A) Mastitis is characterized by shooting pain between feedings, often radiating to the chest wall. B) Mastitis is characterized by late-onset nipple pain. C) The skin of the affected breast becomes pink, flaking, and pruritic. D) Nipple soreness when the infant latches on is not a symptom of mastitis. E) The pain from mastitis does not radiate to the underarm area.

15.A postpartal client recovering from deep vein thrombosis is being discharged. What areas of teaching on self-care and anticipatory guidance should the nurse discuss with the client? Note: Credit will be given only if all correct choices and no incorrect choices are selected. Select all that apply. A) Avoid crossing the legs. B) Avoid prolonged standing or sitting. C) Take frequent walks. D) Take a daily aspirin dose of 650 mg. E) Avoid long car trips.

Answer: A, B, C Explanation: A) Women should be taught to avoid prolonged standing or sitting in one position or sitting with legs crossed. B) Women should be taught to avoid prolonged standing or sitting in one position or sitting with legs crossed. C) Women should be advised to avoid a sedentary lifestyle and to exercise as much as possible (walking is ideal). D) Taking a daily aspirin increases anticoagulant activity, and should be avoided if the client is being treated with other anticoagulants. E) Avoiding long car trips is not necessary. The client should be encouraged to take frequent breaks during long car trips and to walk around, thereby preventing prolonged venous stasis.

23)A newborn who has not voided by 48 hours after birth should be assessed for which of the following? Note: Credit will be given only if all correct choices and no incorrect choices are selected. Select all that apply. A) Restlessness B) Pain C) Kidney distention D) Adequacy of fluid intake E) Lethargy

Answer: A, B, D Explanation: A) A newborn who has not voided by 48 hours after birth should be assessed for restlessness. B) A newborn who has not voided by 48 hours after birth should be assessed for pain. C) A newborn who has not voided by 48 hours after birth should be assessed for bladder distention, not kidney distention. D) A newborn who has not voided by 48 hours after birth should be assessed for adequacy of fluid intake. E) A newborn who has not voided by 48 hours after birth should be assessed for restlessness, not lethargy.

28)The nurse is providing care to a couple who have experienced a perinatal loss. Which is an expected early physical response to the loss? A) Confusion B) Preoccupation C) Loss of appetite D) Poor concentration

Answer:C Explanation: A) Confusion is an early cognitive, not physical, response to the loss. B) Preoccupation is an early cognitive, not physical, response to the loss. C) Loss of appetite is an early physical response to loss. D) Poor concentration is an early cognitive, not physical, response to loss.

5) Marked changes that occur in the cardiopulmonary system at birth include which of the following? Note: Credit will be given only if all correct choices and no incorrect choices are selected. Select all that apply. A) Closure of the foramen ovale B) Closure of the ductus venosus C) Mean blood pressure of 31 to 61 mmHg in full-term resting newborns D) Increased systemic vascular resistance and decreased pulmonary vascular resistance E) Opening of the ductus arteriosus

Answer: A, B, D Explanation: A) Closure of the foramen ovale is a function of changing arterial pressures. B) Closure of the ductus venosus is related to mechanical pressure changes that result from severing the cord, redistribution of blood, and cardiac output. C) The average mean blood pressure of 31 to 61 mmHg in full-term resting newborns is a normal finding, but not a marked change in the cardiopulmonary system. D) Increased systemic vascular resistance and decreased pulmonary vascular resistance; with the loss of the low-resistance placenta, systemic vascular resistance increases, resulting in greater systemic pressure. The combination of vasodilation and increased pulmonary blood flow decreases pulmonary vascular resistance. E) Functional closure, not opening, of the ductus arteriosus in the well newborn starts at 10 to 15 hours after birth.

2) The nurse tells a mother that the doctor is preparing to circumcise her newborn. The mother expresses concern that the infant will be uncomfortable during the procedure. The nurse explains that the physician will numb the area before the procedure. Additional methods of comfort often used during the procedure include which of the following? Note: Credit will be given only if all correct choices and no incorrect choices are selected. Select all that apply. A) Providing a pacifier B) Stroking the head C) Restraining both arms and legs D) Talking to the infant E) Giving the infant a sedative before the procedure

Answer: A, B, D Explanation: A) Providing a pacifier is an accepted method of soothing during the circumcision. B) Stroking the head is an accepted method of soothing during the circumcision. C) Only the legs are restrained during circumcision. D) Talking to the infant is an accepted method of soothing during the circumcision. E) The infant is never given a sedative.

10.Which findings would indicate the presence of a perineal wound infection? Note: Credit will be given only if all correct choices and no incorrect choices are selected. Select all that apply. A) Redness B) Tender at the margins C) Vaginal bleeding D) Hardened tissue E) Purulent drainage

Answer: A, B, D, E Explanation: A) Redness is a classic sign of a perineal wound infection. B) The wound is typically red, indurated, tender at the margins, and draining purulent exudate. C) Vaginal bleeding is nonspecific to identifying a perineal wound infection. D) The wound is typically red, indurated, tender at the margins, and draining purulent exudate. E) Purulent drainage is a classic sign of a perineal wound infection.

11)The nurse is caring for a newborn with jaundice. The parents question why the newborn is not under phototherapy lights. The nurse explains that the fiber-optic blanket is beneficial because of which of the following? Note: Credit will be given only if all correct choices and no incorrect choices are selected. Select all that apply. A) Lights can stay on all the time. B) The eyes do not need to be covered. C) The lights will need to be removed for feedings. D) Newborns do not get overheated. E) Weight loss is not a complication of this system.

Answer: A, B, D, E Explanation: A) With the fiber-optic blanket, the light stays on at all times. B) The eyes do not have to be covered with a fiber optic blanket. C) With the fiber-optic blanket, the light stays on at all times, and the newborn is accessible for care, feeding, and diaper changes. D) With the fiber-optic blanket, greater surface area is exposed and there are no thermoregulation issues. E) Fluid and weight loss are not complications of fiber-optic blankets

26.Risk factors associated with increased risk of thromboembolic disease include which of the following? Note: Credit will be given only if all correct choices and no incorrect choices are selected. Select all that apply. A) Diabetes mellitus B) Varicose veins C) Hypertension D) Adolescent pregnancy E) Malignancy

Answer: A, B, E Explanation: A) Diabetes mellitus is a risk factor for thromboembolic disease. B) Varicose veins are a risk factor for thromboembolic disease. C) Hypertension is not a risk factor for thromboembolic disease. D) Advanced maternal age, not adolescence, is a risk factor for thromboembolic disease. E) Malignancy is a risk factor for thromboembolic disease

13)The nurse is administering erythromycin (Ilotycin) ointment to a newborn. What factors are associated with administration of this medication? Note: Credit will be given only if all correct choices and no incorrect choices are selected. Select all that apply. A) The medication should be instilled in the lower conjunctival sac of each eye. B) The eyelids should be massaged gently to distribute the ointment. C) The medication must be given immediately after delivery. D) The medication does not cause any discomfort to the infant. E) The medication can interfere with the baby's ability to focus.

Answer: A, B, E Explanation: A) Successful eye prophylaxis requires that the medication be instilled in the lower conjunctival sac of each eye. B) After administration, the nurse massages the eyelid gently to distribute the ointment. C) Instillation may be delayed up to 1 hour after birth to allow eye contact during parent-newborn bonding. D) Eye prophylaxis medication can cause chemical conjunctivitis, which gives the newborn some discomfort and may interfere with the baby's ability to focus on the parents' faces and can result in edema, inflammation, and discharge. E) Eye prophylaxis medication can cause chemical conjunctivitis, which gives the newborn some discomfort and can interfere with the baby's ability to focus on the parents' faces.

9) To maintain a healthy temperature in the newborn, which of the following actions should be taken? Note: Credit will be given only if all correct choices and no incorrect choices are selected. Select all that apply. A) Keep the newborn's clothing and bedding dry. B) Reduce the newborn's exposure to drafts. C) Do not use the radiant warmer during procedures. D) Do not wrap the newborn. E) Encourage the mother to snuggle with the newborn under blankets.

Answer: A, B, E Explanation: A) To maintain a healthy temperature in the newborn, keep the newborn's clothing and bedding dry. B) To maintain a healthy temperature in the newborn, reduce the newborn's exposure to drafts. C) To maintain a healthy temperature in the newborn, use the radiant warmer during procedures. D) To maintain a healthy temperature in the newborn, double-wrap the newborn. E) To maintain a healthy temperature in the newborn, encourage the mother to snuggle with the newborn under blankets.

3. A client is experiencing excessive bleeding immediately after the birth of her newborn. After speeding up the IV fluids containing oxytocin, with no noticeable decrease in the bleeding, the nurse should anticipate the physician requesting which medications? Note: Credit will be given only if all correct choices and no incorrect choices are selected. Select all that apply. A) Methergine B) Coumadin C) Misoprostol D) Serotonin reuptake inhibitors (SSRIs) E) Nonsteroidal anti-inflammatory drugs

Answer: A, C Explanation: A) Methergine is commonly used orally for postpartum hemorrhage. B) Coumadin (warfarin) is an anticoagulant and is not used for postpartum hemorrhage. C) Misoprostol is commonly used rectally for postpartum hemorrhage. D) Serotonin reuptake inhibitors (SSRIs) are antidepressants and would not be used for postpartum hemorrhage. E) Nonsteroidal anti-inflammatory drugs increase anticoagulant activity and would not be used for postpartum hemorrhage.

9) A postpartum mother questions whether the environmental temperature should be warmer in the baby's room at home. The nurse responds that the environmental temperature should be warmer for the newborn. This response is based on which newborn characteristics that affect the establishment of thermal stability? Note: Credit will be given only if all correct choices and no incorrect choices are selected. Select all that apply. A) Newborns have less subcutaneous fat than do adults. B) Infants have a thick epidermis layer. C) Newborns have a large body surface to weight ratio. D) Infants have increased total body water. E) Newborns have more subcutaneous fat than do adults.

Answer: A, C, D Explanation: A) Heat transfer from neonatal organs to skin surface is increased compared to adults due to the neonate's decreased subcutaneous fat. B) Preterm infants have increased heat loss via evaporation due to thin skin. C) Heat transfer from neonatal organs to skin surface is increased compared to adults due to the neonate's large body surface to weight ratio. D) Preterm infants have increased heat loss via evaporation due to increased total body water. E) Newborns do not have more subcutaneous fat than adults.

4) The nurse initiates newborn admission procedures and evaluates the newborn's need to remain under observation by assessing which of the following? Note: Credit will be given only if all correct choices and no incorrect choices are selected. Select all that apply. A) Respiratory rate B) Skin texture C) Airway clearance D) Ability to feed E) Head weight

Answer: A, C, D Explanation: A) The nurse initiates newborn admission procedures and evaluates the newborn's need to remain under observation by assessing vital signs (body temperature, heart rate, respiratory rate). B) The nurse initiates newborn admission procedures and evaluates the newborn's need to remain under observation by assessing skin color, not skin texture. C) The nurse initiates newborn admission procedures and evaluates the newborn's need to remain under observation by assessing airway clearance. D) The nurse initiates newborn admission procedures and evaluates the newborn's need to remain under observation by assessing ability to feed. E) The nurse initiates newborn admission procedures and evaluates the newborn's need to remain under observation by assessing circumference and body weight, not head weight.

26)The nurse is teaching the parents of a newborn who has been exposed to HIV how to care for the newborn at home. Which instructions should the nurse emphasize? Note: Credit will be given only if all correct choices and no incorrect choices are selected. Select all that apply. A) Use proper hand-washing technique. B) Provide three feedings per day. C) Place soiled diapers in a sealed plastic bag. D) Cleanse the diaper changing area with a 1:10 bleach solution after each diaper change. E) Take the temperature rectally.

Answer: A, C, D Explanation: A) The nurse should instruct the parents on proper hand-washing technique. B) Small, frequent meals are recommended. C) The nurse should instruct parents to that soiled diapers are to be placed in plastic bags, sealed, and disposed of daily. D) The nurse should instruct parents that the diaper-changing areas should be cleaned with a 1:10 dilution of household bleach after each diaper change. E) Taking rectal temperatures is to be avoided because it could stimulate diarrhea.

8) A nurse is instructing nursing students about the procedure for vitamin K administration. What information should be included? Note: Credit will be given only if all correct choices and no incorrect choices are selected. Select all that apply. A) Gently massage the site after injection. B) Use a 22-gauge, 1-inch needle. C) Inject in the vastus lateralis muscle. D) Cleanse the site with alcohol prior to injection. E) Inject at a 45-degree angle.

Answer: A, C, D Explanation: A) The nurse would remove the needle and massage the site with an alcohol swab. B) Vitamin K is given IM using a 25-gauge, 5/8-inch needle. C) Vitamin K is given intramuscularly in the vastus lateralis muscle. D) Before injecting, the nurse must thoroughly clean the newborn's skin site for the injection with a small alcohol swab. E) Vitamin K is given IM at a 90-degree angle.

27)Which of the following are important behaviors to assess in the neurologic assessment? Note: Credit will be given only if all correct choices and no incorrect choices are selected. Select all that apply. A) State of alertness B) Active posture C) Quality of muscle tone D) Cry E) Motor activity

Answer: A, C, D, E Explanation: A) Important behaviors to assess are the state of alertness, resting posture, cry, and quality of muscle tone and motor activity. B) Resting posture is assessed, not active posture. C) Important behaviors to assess are the state of alertness, resting posture, cry, and quality of muscle tone and motor activity. D) Important behaviors to assess are the state of alertness, resting posture, cry, and quality of muscle tone and motor activity. E) Important behaviors to assess are the state of alertness, resting posture, cry, and quality of muscle tone and motor activity.

23)The nurse should inform the parents of a newborn that they should call their healthcare provider when which of the following occurs? Note: Credit will be given only if all correct choices and no incorrect choices are selected. Select all that apply. A) Continual rise in temperature B) Decreased frequency of stools C) Absence of breathing longer than 20 seconds D) Lethargy E) Refusal of two feedings in a row

Answer: A, C, D, E Explanation: A) Parents should call their healthcare provider due to a continual rise in temperature. B) Parents should call their healthcare provider when there are two consecutive green watery or black stools or increased frequency of stools. C) Parents should call their healthcare provider in the absence of breathing longer than 20 seconds. D) Parents should call their healthcare provider if the newborn exhibits lethargy and listlessness. E) Parents should call their healthcare provider if the newborn has refused two feedings in a row.

23)Antibiotics have been ordered for a newborn with an infection. Which interventions would the nurse prepare to implement? Note: Credit will be given only if all correct choices and no incorrect choices are selected. Select all that apply. A) Obtain skin cultures. B) Restrict parental visits. C) Evaluate bilirubin levels. D) Administer oxygen as ordered. E) Observe for signs of hypoglycemia.

Answer: A, C, D, E Explanation: A) The nurse will assist in obtaining skin cultures. Skin cultures are taken of any lesions or drainage from lesions or reddened areas. B) Restricting parental visits has not been shown to have any effect on the rate of infection and may be harmful to the newborn's psychologic development. C) The nurse will observe for hyperbilirubinemia, anemia, and hemorrhagic symptoms. D) The nurse will administer oxygen as ordered. E) ) The nurse will observe for signs of hypoglycemia.

20)A new mother is concerned because the anterior fontanelle swells when the newborn cries. Explaining normal findings concerning the fontanelles, the nurse states which of the following? Note: Credit will be given only if all correct choices and no incorrect choices are selected. Select all that apply. A) The fontanelles can swell with crying. B) The fontanelles might be depressed. C) The fontanelles can pulsate with the heartbeat. D) The fontanelles might bulge. E) The fontanelles can swell when stool is passed.

Answer: A, C, E Explanation: A) Newborn fontanelles can swell when the newborn cries. B) Depressed fontanelles indicate dehydration. C) Newborn fontanelles can pulsate with the heartbeat. D) Bulging fontanelles signify increased intracranial pressure. E) Newborn fontanelles can swell when the newborn passes a stool.

24)The nurse is caring for the newborn of a drug-addicted mother. Which assessment findings would be typical for this newborn? Note: Credit will be given only if all correct choices and no incorrect choices are selected. Select all that apply. A) Hyperirritability B) Decreased muscle tone C) Exaggerated reflexes D) Low pitched cry E) Transient tachypnea

Answer: A, C, E Explanation: A) Newborns born to drug-addicted mothers exhibit hyperirritability. B) Newborns born to drug-addicted mothers show increased, not decreased, muscle tone. C) Newborns born to drug-addicted mothers exhibit exaggerated reflexes. D) Newborns born to drug-addicted mothers exhibit a high-pitched, not a low- pitched, cry. E) Newborns born to drug-addicted mothers exhibit transient tachypnea.

34)The newborn's cry should have which of the following characteristics? Note: Credit will be given only if all correct choices and no incorrect choices are selected. Select all that apply. A) Medium pitch B) Shrillness C) Strength D) High pitch E) Lusty

Answer: A, C, E Explanation: A) The newborn's cry should be strong, lusty, and of medium pitch. B) A high-pitched, shrill cry is abnormal and may indicate neurologic disorders or hypoglycemia. C) The newborn's cry should be strong, lusty, and of medium pitch. D) A high-pitched, shrill cry is abnormal and may indicate neurologic disorders or hypoglycemia. E) The newborn's cry should be strong, lusty, and of medium pitch

3) A new mother is concerned about a mass on the newborn's head. The nurse assesses this to be a cephalohematoma based on which characteristics? Note: Credit will be given only if all correct choices and no incorrect choices are selected. Select all that apply. A) The mass appeared on the second day after birth. B) The mass appears larger when the newborn cries. C) The head appears asymmetrical. D) The mass appears on only one side of the head. E) The mass overrides the suture line.

Answer: A, D Explanation: A) A cephalohematoma is a collection of blood resulting from ruptured blood vessels between the surface of a cranial bone and the periosteal membrane. These areas emerge as defined hematomas between the first and second days. B) A cephalohematoma does not increase in size when the newborn cries. C) Molding causes the head to appear asymmetrical because of the overriding of cranial bones during labor and birth. D) Cephalohematomas can be unilateral or bilateral, but do not cross the suture lines. E) Cephalohematomas can be unilateral or bilateral, but do not cross the suture lines.

2) The nurse is caring for a postpartum client who is experiencing afterpains following the birth of her third child. Which comfort measure should the nurse implement to decrease her pain? Note: Credit will be given only if all correct choices and no incorrect choices are selected. Select all that apply. A) Offer a warm water bottle for her abdomen. B) Call the physician to report this finding. C) Inform her that this is not normal, and she will need an oxytocic agent. D) Administer a mild analgesic to help with breastfeeding. E) Administer a mild analgesic at bedtime to ensure rest.

Answer: A, D, E Explanation: A) A warm water bottle placed against the low abdomen may reduce the discomfort of afterpains. B) It is not necessary to report this finding to the physician. C) Afterpains are not abnormal in multiparas. Oxytocic agents stimulate uterine contraction and increase the discomfort of the afterpains. D) The breastfeeding mother may find it helpful to take a mild analgesic agent approximately 1 hour before feeding her infant. E) An analgesic agent such as ibuprofen is also helpful at bedtime if the afterpains interfere with the mother's rest.

29)Many newborns exposed to HIV/AIDS show signs and symptoms of disease within days of birth that include which of the following? Note: Credit will be given only if all correct choices and no incorrect choices are selected. Select all that apply. A) Swollen glands B) Hard stools C) Smaller than average spleen and liver D) Rhinorrhea E) Interstitial pneumonia

Answer: A, D, E Explanation: A) Signs that may be seen in the early infancy period include swollen glands. B) Signs that may be seen in the early infancy period include recurrent gastrointestinal (GI) problems that include diarrhea. C) Signs that may be seen in the early infancy period include enlarged spleen and liver. D) Signs that may be seen in the early infancy period include rhinorrhea. E) Signs that may be seen in the early infancy period include interstitial pneumonia

27)A mother who is HIV-positive has given birth to a term female. What plan of care is most appropriate for this infant? A) Test with an HIV serologic test at 8 months. B) Begin prophylactic AZT (Zidovudine) administration. C) Provide 4 to 5 large feedings throughout the day. D) Encourage the mother to breastfeed the child.

Answer: B Explanation: A) Currently available HIV serologic tests (enzyme-linked immunosorbent assay [ELISA] and Western blot test) cannot distinguish between maternal and infant antibodies; therefore, they are inappropriate for infants up to 18 months of age. B) For infants, AZT is started prophylactically 2 mg/kg/dose PO every 6 hours beginning as soon after birth as possible and continuing for 6 weeks. C) Nutrition is essential because failure to thrive and weight loss are common. Small, frequent feedings and food supplementation are helpful. D) Breastfeeding should be avoided with an HIV-positive mother, as transmission of the HIV virus to the newborn in breast milk is well documented.

13)The nurse is preparing an educational in-service presentation about jaundice in the newborn. What content should the nurse include in this presentation? Note: Credit will be given only if all correct choices and no incorrect choices are selected. Select all that apply. A) Physiologic jaundice occurs after 24 hours of age. B) Pathologic jaundice occurs after 24 hours of age. C) Phototherapy increases serum bilirubin levels. D) The need for phototherapy depends on the bilirubin level and age of the infant. E) Kernicterus causes irreversible neurological damage.

Answer: A, E Explanation: A) Physiologic or neonatal jaundice is a normal process that occurs during transition from intrauterine to extrauterine life, and appears after 24 hours of life. B) Diagnosis of pathologic jaundice is given to newborns who exhibit jaundice within the first 24 hours of life. C) Phototherapy decreases serum bilirubin levels. D) The decision to start phototherapy is based on two factors: gestational age and age in hours. E) Kernicterus refers to the deposition of unconjugated bilirubin in the basal ganglia of the brain and to permanent neurologic sequelae of untreated hyperbilirubinemia

11)The nurse assesses a sleeping 1-hour-old, 39-weeks'-gestation newborn. The assessment data that would be of greatest concern would be which of the following? A) Temperature 97.9°F B) Respirations 68 breaths/minute C) Stable vital signs 45 minutes ago D) Heart rate 156 beats/min

Answer: B Explanation: A) 97.9°F is within the normal temperature range of 97.5-99°F. B) The normal respiratory rate is 30-60 breaths/min; 68 breaths/min could represent a less-than-ideal transition. C) The vital signs for a healthy term newborn should be monitored at least every 30 minutes until the newborn's condition has remained stable for 2 hours. D) This heart rate is within the normal range of 110-160 beats/min.

11.A postpartum woman is at increased risk for developing urinary tract problems because of which of the following? A) Decreased bladder capacity B) Inhibited neural control of the bladder following the use of anesthetic agents C) Increased bladder sensitivity D) Abnormal postpartum diuresis

Answer: B Explanation: A) A postpartum woman is at increased risk for developing urinary tract problems because of increased bladder capacity. B) A postpartum woman is at increased risk for developing urinary tract problems because of inhibited neural control of the bladder following the use of anesthetic agents. C) A postpartum woman is at increased risk for developing urinary tract problems because of decreased bladder sensitivity from stretching or trauma. D) A postpartum woman is at increased risk for developing urinary tract problems because of normal postpartum diuresis.

14.To prevent the spread of infection, the nurse teaches the postpartum client to do which of the following? A) Address pain early B) Change peri-pads frequently C) Avoid overhydration D) Report symptoms of uterine cramping

Answer: B Explanation: A) Addressing pain early would not be a preventive action for infection. B) Changing peri-pads frequently decreases skin contact with a moist medium that favors bacteria growth. C) Avoiding overhydration actually would increase the risk for infection by not providing adequate fluids to flush the kidneys and bladder. D) Reporting symptoms of uterine cramping would not be a preventive action for infection.

19)The nurse suspects clubfoot in the newborn and assesses for the condition by doing which of the following? A) Adducting the foot and listening for a click. B) Moving the foot to midline and determining resistance. C) Extending the foot and observing for pain. D) Stimulating the sole of the foot.

Answer: B Explanation: A) Adducting the foot and listening for a click is not a typical assessment. B) Clubfoot is suspected when the foot does not turn to a midline position or align readily. C) Extending the foot and observing for pain does not confirm or rule out clubfoot. D) Stimulating the sole of the foot elicits the plantar grasp reflex, and is not an appropriate assessment for clubfoot.

29)The nurse is working with a mother who has just delivered her third child at 33 weeks' gestation. The mother says to the nurse, "This baby doesn't turn her head and suck like the older two children did. Why?" What is the best response by the nurse? A) "Every baby is different. This is just one variation of normal that we see on a regular basis." B) "This baby might not have a rooting or sucking reflex because she is premature." C) "When she is wide awake and alert, she will probably root and suck even if she is early." D) "She might be too tired from the birthing process and need a couple of days to recover."

Answer: B Explanation: A) Although each baby is unique and different from siblings, this response is not accurate. B) Preterm babies may have suppressed or absent root and suck reflexes. C) This statement is true of term infants, but this infant is preterm. D) Although birth is stressful even to term newborns, and some require a day or two of recovery to become fully alert, this infant is preterm.

26)One day after giving birth vaginally, a client develops painful vesicular lesions on her perineum and vulva. She is diagnosed with a primary herpes simplex 2 infection. What is the expected care for her neonate? A) Meticulous hand washing and antibiotic eye ointment administration. B) Intravenous acyclovir (Zovirax) and contact precautions. C) Cultures of blood and CSF and serial chest x-rays every 12 hours. D) Parental rooming-in and four intramuscular injections of penicillin.

Answer: B Explanation: A) Although meticulous hand washing by staff and parents is important, antibiotic eye ointment is used for conjunctivitis of gonorrhea or chlamydia. B) Administering intravenous acyclovir (Zovirax) and contact precautions are appropriate measures for an infant at risk for developing herpes simplex 2 infection. C) Cultures of blood and CSF cultures are appropriate, but chest X-rays are not indicated. Chest X-rays are obtained if the neonate is thought to have group B strep pneumonia. D) Parental rooming-in is encouraged, but penicillin does not treat viral illness

32)A new mother is holding her 2-hour-old son. The delivery occurred on the due date. His Apgar score was 9 at both 1 and 5 minutes. The mother asks the nurse why her son was so wide awake right after birth, and now is sleeping so soundly. What is the nurse's best response? A) "Don't worry. Babies go through a lot of these little phases." B) "Your son is in the sleep phase. He'll wake up soon." C) "Your son is exhausted from being born, and will sleep 6 more hours." D) "Your breastfeeding efforts have caused excessive fatigue in your son."

Answer: B Explanation: A) Although this infant's behavior is expected, nurses must avoid using clichés in therapeutic communication. B) The first period of reactivity lasts approximately 30 minutes after birth. During this period the newborn is awake and active and may appear hungry and have a strong sucking reflex. After approximately half an hour, the newborn's activity gradually diminishes, and the heart rate and respirations decrease as the newborn enters the sleep phase. The sleep phase may last from a few minutes to 2 to 4 hours. C) Six hours of sleep at this point is not an expected finding. D) Breastfeeding does not cause fatigue in a normal term newborn.

15)The client with blood type O Rh-negative has given birth to an infant with blood type O Rh-positive. The infant has become visibly jaundiced at 12 hours of age. The mother asks why this is happening. What is the best response by the nurse? A) "The RhoGAM you received at 28 weeks' gestation did not prevent alloimmunization." B) "Your body has made antibodies against the baby's blood that are destroying her red blood cells." C) "The red blood cells of your baby are breaking down because you both have type O blood." D) "Your baby's liver is too immature to eliminate the red blood cells that are no longer needed."

Answer: B Explanation: A) Although this statement is true, the term "alloimmunization" is not likely to be understood by the client. It is better to explain what is happening using more understandable terminology. B) This explanation is accurate and easy for the client to understand. Newborns of Rh-negative and O blood type mothers are carefully assessed for blood type status, appearance of jaundice, and levels of serum bilirubin. C) Mother and baby's both having type O blood is not a problem. ABO incompatibility occurs if mother is O and baby is A or B. D) The infant's liver is indeed too immature to eliminate red blood cells, but the hemolysis from the maternal antibodies is the cause of the jaundice

7) The nurse expects an initial weight loss for the average postpartum client to be which of the following? A) 5 to 8 pounds B) 10 to 12 pounds C) 12 to 15 pounds D) 15 to 20 pounds

Answer: B Explanation: A) An initial weight loss of 10 to 12 lbs. occurs as a result of the birth of infant, placenta, and amniotic fluid. B) An initial weight loss of 10 to 12 lbs. occurs as a result of the birth of infant, placenta, and amniotic fluid. C) An initial weight loss of 10 to 12 lbs. occurs as a result of the birth of infant, placenta, and amniotic fluid. D) An initial weight loss of 10 to 12 lbs. occurs as a result of the birth of infant, placenta, and amniotic fluid.

10)Every time the nurse enters the room of a postpartum client who gave birth 3 hours ago, the client asks something else about her birth experience. What action should the nurse take? A) Answer questions quickly and try to divert her attention to other subjects. B) Review the documentation of the birth experience and discuss it with her. C) Contact the physician to warn him the client might want to file a lawsuit, based on her preoccupation with the birth experience. D) Submit a referral to Social Services because of possible obsessive behavior

Answer: B Explanation: A) Answering questions quickly and trying to divert the client's attention trivializes her questions and does not allow her to sort out the reality from her fantasized experience. B) The client may talk about her labor and birth experience. The nurse should provide opportunities to discuss the birth experience in a nonjudgmental atmosphere if the woman desires to do so. C) Asking questions about the birth experience is normal behavior. Contacting the physician is not warranted. D) Submitting a referral to Social Services is inappropriate, because this client's behavior is normal.

2. The nurse is assisting a multiparous woman to the bathroom for the first time since her delivery 3 hours ago. When the client stands up, blood runs down her legs and pools on the floor. The client turns pale and feels weak. What would be the first action of the nurse? A) Assist the client to empty her bladder B) Help the client back to bed to check the fundus C) Assess her blood pressure and pulse D) Begin an IV of lactated Ringer's solution

Answer: B Explanation: A) Assisting the client to empty her bladder is not the first action the nurse would take. B) Massaging the fundus is the top priority because of the excessive blood loss. If the fundus is not firm, gentle fundal massage is performed until the uterus contracts. C) Blood pressure and pulse do not change until 1000 to 2000 mL of blood has been lost. D) An IV might need to be started if the client becomes symptomatic

21)Placing the baby at mother's breast facilitates early latch and promotes successful breastfeeding. When should breastfeeding be initiated? A) 6 to 12 hours after birth B) Within 1 hour of birth C) 24 hours after birth D) 48 hours after birth

Answer: B Explanation: A) Breastfeeding should be initiated within the first hour of life, not 6 to 12 hours after birth. B) Breastfeeding should be initiated within the first hour of life unless medically contraindicated. C) Breastfeeding should be initiated within the first hour of life, not 24 hours after birth. D) Breastfeeding should be initiated within the first hour of life, not 48 hours after birth.

13)The nurse is completing the gestational age assessment on a newborn while in the mother's postpartum room. During the assessment, the mother asks what aspects of the baby are being checked. What is the nurse's best response? A) "I'm checking to make sure the baby has all of its parts." B) "This assessment looks at both physical aspects and the nervous system." C) "This assessment checks the baby's brain and nerve function." D) "Don't worry. We perform this check on all the babies."

Answer: B Explanation: A) Clinical gestational age assessment tools have two components: external physical characteristics and neurologic or neuromuscular development evaluations. B) Clinical gestational age assessment tools have two components: external physical characteristics and neurologic or neuromuscular development evaluations. C) Clinical gestational age assessment tools have two components: external physical characteristics and neurologic or neuromuscular development evaluations. D) Nurses must always use therapeutic communication and giving a "don't worry" answer dismisses the client's question or concern

14)The nurse notes that a 36-hour-old newborn's serum bilirubin level has increased from 14 mg/dL to 16.6 mg/dL in an 8-hour period. What nursing intervention would be included in the plan of care for this newborn? A) Continue to observe B) Begin phototherapy C) Begin blood exchange transfusion D) Stop breastfeeding

Answer: B Explanation: A) Continued observation is only appropriate with normal findings. B) Neonatal hyperbilirubinemia must be considered pathologic if the serum bilirubin concentration is rising by more than 0.2 mg/dL per hour. If the newborn is over 24 hours old, which is past the time where an increase in bilirubin would result from pathologic causes, phototherapy may be the treatment of choice to prevent the possible complications of kernicterus. C) If a newborn has hemolysis with an unconjugated bilirubin level of 14 mg/dL, weighs less than 2500 g (birth weight), and is 24 hours old or less, an exchange transfusion may be the best management. This newborn is 36-hours-old. D) The newborn may continue to breastfeed.

9. Which of the following would be considered a clinical sign of hemorrhage? A) Increased blood pressure B) Increasing pulse C) Increased urinary output D) Hunger

Answer: B Explanation: A) Decreased blood pressure would be considered a clinical sign of hemorrhage. B) Increasing pulse, widening pulse pressure would be considered a clinical sign of hemorrhage. C) Decreased urinary output would be considered a clinical sign of hemorrhage. D) Thirst, not hunger, would be considered a clinical sign of hemorrhage.

12)The nurse is caring for a jaundiced infant receiving bank light phototherapy in an isolette. Which finding requires an immediate intervention? A) Eyes are covered, no clothing on, diaper in place B) Axillary temperature 99.7°F C) Infant removed from the isolette for breastfeeding D) Loose bowel movement

Answer: B Explanation: A) Eye coverings are used because it is not known if phototherapy injures delicate eye structures, particularly the retina. Because the tissue absorbs the light, best results are obtained when there is maximum skin surface exposure. B) Temperature assessment is indicated to detect hypothermia or hyperthermia. Normal temperature ranges are 97.7°F-98.6°F. Vital signs should be monitored every 4 hours with axillary temperatures. C) Breastfeeding should continue during phototherapy; removing the infant for feedings repositions the infant to prevent pressure areas. D) Infants undergoing phototherapy treatment have increased water loss and loose stools as a result of bilirubin excretion.

26)The community nurse is meeting a new mother for the first time. The client delivered her first child 5 days ago after a 12-hour labor. Neither the mother nor the infant had any complications during the birth or postpartum period. Which statement by the client would indicate to the nurse that the client is experiencing postpartum blues? A) "I am so happy and blessed to have my new baby." B) "One minute I'm laughing and the next I'm crying." C) "My husband is helping out by changing the baby at night." D) "Breastfeeding is going quite well now that the engorgement is gone.

Answer: B Explanation: A) Feeling happy is not a symptom of postpartum blues. B) The postpartum blues consist of a transient period of depression that occurs during the first few days of puerperium. Symptoms may include mood swings, anger, weepiness, anorexia, difficulty sleeping, and a feeling of letdown. C) The husband's assistance does not relate to postpartum blues. D) Successful lactation is not a symptom of postpartum blues.

13.The nurse is calling clients at 4 weeks postpartum. Which of the following clients should be seen immediately? A) The client who describes feeling sad all the time B) The client who reports hearing voices talking about the baby C) The client who states she has no appetite and wants to sleep all day D) The client who says she needs a refill on her sertraline (Zoloft) next week

Answer: B Explanation: A) Feeling sad is an indication the client is experiencing postpartum blues, and is not the highest priority. B) Hearing voices is an indication the client is experiencing postpartum psychosis, and is the highest priority because the voices might tell her to harm her baby. C) Having no appetite and sleeping all day is an indication the client is experiencing postpartum depression, but is not the highest priority. D) A client on medications needs refills on time, but right now she has medication, and therefore is not a high priority.

8) In utero, what is the organ responsible for gas exchange? A) Umbilical vein B) Placenta C) Inferior vena cava D) Right atrium

Answer: B Explanation: A) From the placenta, highly oxygenated blood flows through the umbilical vein. B) In utero, the placenta is the organ of gas exchange. C) From the placenta, highly oxygenated blood flows through the umbilical vein. A small amount of blood perfuses the liver, with the majority of blood volume flowing through the inferior vena cava and to the right atrium. D) From the placenta, highly oxygenated blood flows through the umbilical vein. A small amount of blood perfuses the liver, with the majority of blood volume flowing through the inferior vena cava and to the right atrium.

30)The student nurse notices that the newborn seems to focus on the mother's eyes. The nursing instructor explains that this newborn behavior is which of the following? A) Habituation B) Orientation C) Self-quieting D) Reactivity

Answer: B Explanation: A) Habituation is the newborn's ability to process and respond to complex stimulation. B) Orientation is the newborn's ability to be alert to, follow, and fixate on complex visual stimuli that have a particular appeal and attraction. The newborn prefers the human face and eyes, and bright shiny objects. C) Self-quieting is the ability of newborns to use their own resources to quiet and comfort themselves. D) The newborn usually shows a predictable pattern of behavior during the first several hours after birth, characterized by two periods of reactivity separated by a sleep phase.

32)The nurse is preparing an educational session on phenylketonuria for a family whose neonate has been diagnosed with the condition. Which statement by a parent indicates that teaching was effective? A) "This condition occurs more frequently among Japanese people." B) "We must be very careful to avoid most proteins to prevent brain damage." C) "Carbohydrates can cause our baby to develop cataracts and liver damage." D) "Our baby's thyroid gland isn't functioning properly.

Answer: B Explanation: A) Japanese people have a very low rate of PKU disease; it is most common among northern Europeans. B) PKU is the inability to metabolize phenylalanine, an amino acid found in most dietary protein sources. Excessive accumulation of phenylalanine and its abnormal metabolites in the brain tissue leads to progressive, irreversible intellectual disability. C) Galactosemia is a carbohydrate metabolism disease. D) Congenital hypothyroidism is the disorder of low thyroid function at birth.

5) The student nurse has performed a gestational age assessment of an infant, and finds the infant to be at 32 weeks. On which set of characteristics is the nurse basing this assessment? A) Lanugo mainly gone, little vernix across the body B) Prominent clitoris, enlarging minora, anus patent C) Full areola, 5 to 10 mm bud, pinkish-brown in color D) Skin opaque, cracking at wrists and ankles, no vessels visible

Answer: B Explanation: A) Lanugo and vernix disappear as the infant approaches term. B) At 30 to 32 weeks' gestation, the clitoris is prominent, and the labia majora are small and widely separated. As gestational age increases, the labia majora increase in size. At 36 to 40 weeks, they nearly cover the clitoris. At 40 weeks and beyond, the labia majora cover the labia minora and clitoris. C) Areolas develop greater size with advancing gestational age. D) The skin of a preterm infant is translucent, and vessels are visible through the skin.

1) The nurse determines the fundus of a postpartum client to be boggy. Initially, what should the nurse do? A) Document the findings. B) Catheterize the client. C) Massage the uterine fundus until it is firm. D) Call the physician immediately.

Answer: C Explanation: A) Documenting the findings would come after massage, reassessment, and evaluation. B) Catheterizing the client might be indicated if assessment reveals a full bladder and inability to void, but not as an initial intervention. C) The nurse would massage the uterine fundus until it is firm by keeping one hand in position and stabilizing the lower portion of the uterus. With one hand used to massage the fundus, the nurse would put steady pressure on the top of the now- firm fundus to see if she was able to express any clots. D) Calling the physician immediately is not necessary until more data are obtained

33)The nurse is teaching a newborn care class to parents who are about to give birth to their first babies. Which statement by a parent indicates that teaching was effective? A) "My baby will be able to focus on my face when she is about a month old." B) "My baby might startle a little if a loud noise happens near him." C) "Newborns prefer sour tastes." D) "Our baby won't have a sense of smell until she is older."

Answer: B Explanation: A) Newborns can focus on faces, eyes, and shiny objects at birth. B) Swaddling, placing a hand on the abdomen, or holding the arms to prevent a startle reflex are ways to soothe the newborn. The settled newborn is then able to attend to and interact with the environment. C) Newborns can distinguish between sweet and sour at 3 days of age. Sugar, for example, increases sucking, and newborns tend to have a preference for sweet tastes. D) Newborns develop the sense of smell rapidly and can differentiate their mother by smell within the first week of life.

25)The nurse is planning care for four infants who were born on this shift. The infant who will require the most detailed assessment is the one whose mother has which of the following? A) A history of obsessive-compulsive disorder (OCD) B) Chlamydia C) Delivered six other children by cesarean section D) A urinary tract infection (UTI)

Answer: B Explanation: A) Obsessive-compulsive disorder (OCD) is not a risk factor for the infant. B) Infants born to mothers with chlamydia infections are at risk for neonatal pneumonia and conjunctivitis, and require close observation of the respiratory status and eyes. C) Having multiple siblings, regardless of how they were delivered, is not a risk factor for the infant. D) An infant whose mother has an untreated urinary tract infection might have been exposed to pathogens, but it is not known whether the mother in this question is on antibiotics.

15)The visiting nurse evaluates a 2-day-old breastfed newborn at home and notes that the baby appears jaundiced. When explaining jaundice to the parents, what would the nurse tell them? A) "Jaundice is uncommon in newborns." B) "Some newborns require phototherapy." C) "Jaundice is a medical emergency." D) "Jaundice is always a sign of liver disease.

Answer: B Explanation: A) Physiologic jaundice is a normal process that can occur after 24 hours of life and develops in more than 60% of term newborns and 80% of preterm neonates. B) Physiologic jaundice is a normal process that can occur after 24 hours of life in about half of healthy newborns. It is not a sign of liver disease. Physiologic jaundice might require phototherapy. C) Physiologic jaundice is a normal process that can occur after 24 hours of life in about half of healthy newborns. It is not a medical emergency. D) Physiologic jaundice is not a sign of liver disease.

21)Mild or chronic anemia in an infant may be treated adequately by which of the following? A) Transfusions with O-negative or typed and cross-matched packed red cells B) Iron supplements or iron-fortified formulas C) Steroid therapy D) Antibiotics or antivirals

Answer: B Explanation: A) Severe cases of anemia are treated with transfusions with O-negative or typed and cross-matched packed red cells. B) Mild or chronic anemia in an infant may be treated adequately with iron supplements or iron-fortified formulas. C) Management of anemia of prematurity includes treating the causative factor (e.g., antibiotics or antivirals used for infection or steroid therapy for disorders of erythrocyte production). D) Management of anemia of prematurity includes treating the causative factor (e.g., antibiotics or antivirals used for infection or steroid therapy for disorders of erythrocyte production).

28)An HIV-positive mother delivered 2 days ago. The infant will be placed in foster care. The nurse is planning discharge teaching for the foster parents on how to care for the newborn at home. Which instructions should the nurse include? A) Do not add food supplements to the baby's diet. B) Place soiled diapers in a sealed plastic bag. C) Wash soiled linens in cool water with bleach. D) Shield the baby's eyes from bright lights.

Answer: B Explanation: A) Small, frequent feedings are recommended, as well as food supplementation as necessary to support weight gain. B) The nurse should instruct the parents about proper hand-washing techniques, proper disposal of soiled diapers, and the importance of wearing gloves when diapering. C) Soiled linens should be washed in hot, sudsy water with bleach. D) Shielding the baby's eyes from bright lights would be recommended for a preterm infant, not an infant with HIV.

5) The nurse has just assisted the father in bathing the newborn 2 hours after birth. The nurse explains that the newborn must remain in the radiant warmer. This is based on which assessment data? A) Heart rate 120 B) Temperature 96.8°F C) Respiratory rate 50 D) Temperature 99.6°F

Answer: B Explanation: A) The heart rate is within normal limits for a newborn 2 hours old. B) The nurse rechecks the temperature after the bath and, if it is stable, dresses the newborn in a shirt, diaper, and cap; wraps the baby; and places the baby in an open crib at room temperature. If the baby's axillary temperature is below 36.5°C (97.7°F), the nurse returns the baby to the radiant warmer. The rewarming process should be gradual to prevent the possibility of hyperthermia. C) The respiratory rate is within normal limits for a newborn 2 hours old. D) This temperature (99.6°F) does not warrant placing the infant back in the radiant warmer.

22)What indications would lead the nurse to suspect sepsis in a newborn? A) Respiratory distress syndrome developing 48 hours after birth B) Temperature drops from 97.4°F to 97.0 2°F hours after 2 hours of warming. C) Irritability and flushing of the skin at 8 hours of age D) Bradycardia and tachypnea developing when the infant is 36 hours old

Answer: B Explanation: A) The infant may deteriorate rapidly in the first 12 to 24 hours after birth if β- hemolytic streptococcal infection is present. B) Temperature instability is often seen with sepsis. Fever is rare in a newborn. C) Irritability or lethargy with pallor after the first 24 hours might indicate sepsis, and the skin is cool and clammy. D) Tachycardia and periods of apnea are seen with sepsis

14)The nurse is caring for a newborn who was recently circumcised. Which nursing intervention is appropriate following the procedure? A) Keep the infant NPO for 4 hours following the procedure. B) Observe for urine output. C) Wrap dry gauze tightly around the penis. D) Clean with cool water with each diaper change.

Answer: B Explanation: A) The newborn does not need to be NPO. B) It is important to observe for the first voiding after a circumcision to evaluate for urinary obstruction related to penile injury and/or edema. C) Gauze should not be wrapped tightly around the penis. Only if bleeding occurs should the nurse apply light pressure with a sterile gauze pad to stop the bleeding within a short time. D) The newborn should be cleaned with warm water with each diaper change.

5) A client in labor is found to have meconium-stained amniotic fluid upon rupture of membranes. At delivery, the nurse finds the infant to have depressed respirations and a heart rate of 80. What does the nurse anticipate? A) Delivery of the neonate on its side with head up, to facilitate drainage of secretions. B) Direct tracheal suctioning by specially trained personnel. C) Preparation for the immediate use of positive pressure to expand the lungs. D) Suctioning of the oropharynx when the newborn's head is delivered

Answer: B Explanation: A) The newborn is not delivered on its side. B) If the infant has absent or depressed respirations, heart rate less than 100 beats/min, or poor muscle tone, direct tracheal suctioning by specially trained personnel is recommended. C) Positive pressure is not used to expand the lungs. D) Current evidence does not support intrapartum oropharyngeal and nasopharyngeal suctioning as they do not prevent or alter the course of MAS.

6) The nurse has received the end-of-shift report on the postpartum unit. Which client should the nurse see first? A) Woman who is 2nd day post-cesarean, moderate lochia serosa B) Woman day of delivery, fundus firm 2 cm above umbilicus C) Woman who had a cesarean section, 1st postpartum day, 4 cm diastasis recti abdominis D) Woman who had a cesarean section, 1st postpartum day, hypoactive bowel sounds all quadrants

Answer: B Explanation: A) This client is not experiencing any unexpected findings. The uterus rids itself of the debris remaining after birth through a discharge called lochia, which is classified according to its appearance and contents. Lochia serosa is a pinkish color. B) This client is the top priority. The fundus should not be positioned above the umbilicus after delivery. If the fundus is in the midline but higher than expected, it is usually associated with clots within the uterus. C) Diastasis recti abdominis is normal and is especially pronounced in women who have undergone a cesarean section. D) Bowel sounds are often decreased after delivery and it may take a few days for the bowel to regain its tone. The woman who has had a cesarean or a difficult birth may benefit from stool softeners.

15)A postpartum mother is concerned that her newborn has not had a stool since birth. The newborn is 18 hours old. What is the nurse's best response? A) "I will call your pediatrician immediately." B) "Passage of the first stool within 48 hours is normal." C) "Your newborn might not have a stool until the third day." D) "Your newborn must be dehydrated."

Answer: B Explanation: A) This is not an emergency situation. B) The first voiding should occur within 24 hours and first passage of stool within 48 hours. C) The passage of the first stool should occur sooner. D) Decreased urinary output and depressed fontanelles indicate dehydration.

14)The nurse is working with a new mother who follows Muslim traditions. Which expectations and actions are appropriate for this client? Note: Credit will be given only if all correct choices and no incorrect choices are selected. Select all that apply. A) Make sure she gets a kosher diet. B) Expect that most visitors will be women. C) Uncover only the necessary skin when assessing. D) The father will take an active role in infant care. E) She will prefer a male physician.

Answer: B, C Explanation: A) A kosher diet is appropriate for Jewish women, not Muslims. Although many of the dietary laws are similar, kosher laws are not appropriate for a Muslim client. B) In Muslim cultures, emphasis on childrearing and infant care activities is on the mother and female relatives. C) Women of the Islamic faith may have specific modesty requirements; the woman must be completely covered, with only her feet and hands exposed. D) In Muslim cultures, emphasis on childrearing and infant care activities is on the mother and female relatives, and there is little involvement of the father in newborn care. E) Women of the Islamic faith may have specific modesty requirements; the woman must be completely covered, with only her feet and hands exposed, and no man, other than the husband or a family member, may be alone with her. Because of the culture of modesty, many Muslim women highly prefer female care providers

24.The postpartum client is suspected of having acute cystitis. Which symptoms would the nurse expect to see in this client? Note: Credit will be given only if all correct choices and no incorrect choices are selected. Select all that apply. A) High fever B) Frequency C) Suprapubic pain D) Chills E) Nausea and vomiting

Answer: B, C Explanation: A) High fever is not usually present in acute cystitis, although it can appear if the cystitis progresses to pyelonephritis. B) Frequency is characteristic of acute cystitis. C) Suprapubic pain is characteristic of acute cystitis. D) Chills are not usually present in acute cystitis, although they can appear if the cystitis progresses to pyelonephritis. E) Nausea and vomiting are not usually present in acute cystitis, although they can appear if the cystitis progresses to pyelonephritis.

10)The nurse is teaching new parents how to dress their newborn. Which statements indicate that teaching has been effective? Note: Credit will be given only if all correct choices and no incorrect choices are selected. Select all that apply. A) "We should keep our home air-conditioned so the baby doesn't overheat." B) "It is important that we dry the baby off as soon as we give him a bath or shampoo his hair." C) "When we change the baby's diaper, we should change any wet clothing or blankets, too." D) "If the baby's body temperature gets too low, he will warm himself up without any shivering." E) "Our baby will have a much faster rate of breathing if he is not dressed warmly enough."

Answer: B, C, D, E Explanation: A) Because of the risk of hypothermia and possible cold stress, minimizing heat loss in the newborn after birth is essential. B) The newborn is particularly prone to heat loss by evaporation immediately after birth and during baths; thus drying the newborn is critical. C) Changing wet clothing or blankets immediately prevents evaporation, one mechanism of heat loss. D) Nonshivering thermogenesis (NST), an important mechanism of heat production unique to the newborn, is the major mechanism through which heat is produced. E) A decrease in the environmental temperature of 2°C is a drop sufficient to double the oxygen consumption of a term newborn and can cause the newborn to show signs of respiratory distress.

17)Which of the following symptoms would be an indication of postpartum blues? Note: Credit will be given only if all correct choices and no incorrect choices are selected. Select all that apply. A) Overeating B) Anger C) Mood swings D) Constant sleepiness E) Crying

Answer: B, C, E Explanation: A) Anorexia, not overeating, would be a symptom of postpartum blues. B) Anger would be a symptom of postpartum blues. C) Mood swings would be a symptom of postpartum blues. D) Difficulty sleeping would be a symptom of postpartum blues. E) Weepiness and crying would be a symptom of postpartum blues.

18)Nursing interventions that foster the process of becoming a mother include which of the following? Note: Credit will be given only if all correct choices and no incorrect choices are selected. Select all that apply. A) Encouraging detachment from the nurse-patient relationship B) Promoting maternal-infant attachment C) Building awareness of and responsiveness to infant interactive capabilities D) Instruct about promoting newborn independence E) Preparing the woman for the maternal social role

Answer: B, C, E Explanation: A) Encouraging interactive therapeutic nurse-patient relationships is a nursing intervention that fosters the process of becoming a mother. B) Promoting maternal-infant attachment is a nursing intervention that fosters the process of becoming a mother. C) Building awareness of and responsiveness to infant interactive capabilities is a nursing intervention that fosters the process of becoming a mother. D) Instructing about infant caregiving is a nursing intervention that fosters the process of becoming a mother. E) Preparing the woman for the maternal social role is a nursing intervention that fosters the process of becoming a mother.

12)A NICU nurse plans care for a preterm newborn that will provide opportunities for development. Which interventions support development in a preterm newborn in a NICU? Note: Credit will be given only if all correct choices and no incorrect choices are selected. Select all that apply. A) Schedule care throughout the day. B) Silence alarms quickly. C) Place a blanket over the top portion of the incubator. D) Do not offer a pacifier. E) Dim the lights.

Answer: B, C, E Explanation: A) Nursing care should be planned to decrease the number of times the baby is disturbed. B) Noise levels can be lowered by replacing alarms with lights or silencing alarms quickly. C) Dimmer switches should be used to shield the baby's eyes from bright lights with blankets over the top portion of the incubator. D) Pacifiers can be offered because they provide opportunities for nonnutritive sucking. E) Dimming the lights may encourage infants to open their eyes and be more responsive to their parents.

26)Which of the following would be considered normal newborn urinalysis values? Note: Credit will be given only if all correct choices and no incorrect choices are selected. Select all that apply. A) Color bright yellow B) Bacteria 0 C) Red blood cells (RBC) 0 D) White blood cells (WBC) more than 4-5/hpf E) Protein less than 5-10 mg/dL

Answer: B, C, E Explanation: A) Urine color should be pale yellow. B) Bacteria value should be 0. C) Red blood cells (RBC) should be 0. D) White blood cells (WBC) should be less than 2-3/hpf. E) Protein less than 5-10 mg/dL would be considered normal

28)During the first several postpartum weeks, the new mother must accomplish certain physical and developmental tasks, including which of the following? Note: Credit will be given only if all correct choices and no incorrect choices are selected. Select all that apply. A) Establish a therapeutic relationship with her physician B) Adapt to altered lifestyles and family structure resulting from the addition of a new member C) Restore her intellectual abilities D) Restore physical condition E) Develop competence in caring for and meeting the needs of her infant

Answer: B, D, E Explanation: A) During the first several postpartum weeks, the new mother must establish a relationship with her new child, not her physician. B) During the first several postpartum weeks, the new mother must adapt to altered lifestyles and family structure resulting from the addition of a new member. C) During the first several postpartum weeks, the new mother must restore her physical condition, not her intellectual abilities. D) During the first several postpartum weeks, the new mother must restore her physical condition. E) During the first several postpartum weeks, the new mother must develop competence in caring for and meeting the needs of her infant

16)Which of the following are considered risk factors for development of severe hyperbilirubinemia? Note: Credit will be given only if all correct choices and no incorrect choices are selected. Select all that apply. A) Northern European descent B) Previous sibling received phototherapy C) Gestational age 27 to 30 weeks D) Exclusive breastfeeding E) Infection

Answer: B, D, E Explanation: A) East Asian or Mediterranean descent is considered a risk factor for development of severe hyperbilirubinemia. B) Previous sibling received phototherapy is considered a risk factor for development of severe hyperbilirubinemia. C) Gestational age 35 to 36 weeks (late preterm gestational age) is considered a risk factor for development of severe hyperbilirubinemia. D) Exclusive breastfeeding, particularly if nursing is not going well and excessive weight loss is experienced, is considered a risk factor for development of severe hyperbilirubinemia. E) Infection is considered a risk factor for development of severe hyperbilirubinemia.

1) The nurse is caring for a newborn 30 minutes after birth. After assessing respiratory function, the nurse would report which findings as abnormal? Note: Credit will be given only if all correct choices and no incorrect choices are selected. Select all that apply. A) Respiratory rate of 66 breaths per minute B) Periodic breathing with pauses of 25 seconds C) Synchronous chest and abdomen movements D) Grunting on expiration E) Nasal flaring

Answer: B, D, E Explanation: A) Immediately after birth and for the next 2 hours, the normal respiratory rate is 60 to 70 breaths per minute. B) Periodic breathing with pauses longer than 20 seconds (apnea) is an abnormal finding that should be reported to the physician. C) Abdominal movements that are synchronous with the chest movements are normal. D) Grunting on expiration is an abnormal finding that should be reported to the physician. E) Nasal flaring is an abnormal finding that should be reported to the physician.

20.The postpartum client has developed thrombophlebitis in her right leg. Which finding requires immediate intervention? A) The client reports she had this condition after her last pregnancy. B) The client develops pain and swelling in her left lower leg. C) The client appears anxious, and describes pressure in her chest. D) The client becomes upset that she cannot go home yet.

Answer: C Explanation: A) A risk factor includes recurrent thromboembolic disease, but this is neither a predictor nor an indication of complications. B) Development of thrombophlebitis is a complication, but not the top priority. C) The most common clinical findings of a pulmonary embolism include dyspnea, pleuritic chest pain, cough with or without hemoptysis, cyanosis, tachypnea and tachycardia, panic, syncope, or sudden hypotension and require immediate intervention. D) Becoming upset is a psychosocial issue and far less important than another finding.

22)Approximately what percentage of the newborn's body weight is water? A) 5% to 10% B) 90% to 95% C) 70% to 75% D) 50% to 60%

Answer: C Explanation: A) During the initial newborn period, term newborns have a physiologic weight loss of about 5% to 10% because fluid shifts. B) Approximately 70% to 75% of the newborn's body weight is water. C) Approximately 70% to 75% of the newborn's body weight is water. D) Approximately 70% to 75% of the newborn's body weight is water.

31)A new father asks the nurse to describe what his baby will experience while sleeping and awake. What is the best response? A) "Babies have several sleep and alert states. Keep watching and you'll notice them." B) "You might have noticed that your child was in an alert awake state for an hour after birth." C) "Newborns have two stages of sleep: deep or quiet sleep and rapid eye movement sleep." D) "Birth is hard work for babies. It takes them a week or two to recover and become more awake."

Answer: C Explanation: A) Although it is true that babies have several sleep and alert states, the wording of this response is condescending and not therapeutic. B) Although this statement is true, it does not respond to the father's question about sleeping now. C) Teaching the parents how to recognize the two sleep stages helps them tune in to their infant's behavioral states. D) Recovery from the birth process only takes a day or two. The newborn usually shows a predictable pattern of behavior during the first several hours after birth

1) The parents of a newborn male ask the nurse whether they should circumcise their son. What is the best response by the nurse? A) "Circumcision should be undertaken to prevent problems in the future." B) "Circumcision might decrease the child's risk of developing a urinary tract infection." C) "Circumcision can sometimes cause complications. What questions do you have?" D) "Circumcision is painful, and should be avoided unless you are Jewish."

Answer: C Explanation: A) Although this is a common reason parents give for requesting circumcision, it is still an opinion not based in medical fact. B) Although circumcision permits exposure of the glans for easier cleaning, getting more information from the parents about their questions or concerns would be better. C) Asking this question allows the nurse to determine what the parents' concerns are, then address them specifically. D) Although circumcision can be painful, most providers administer a penile nerve root block to prevent or minimize procedural pain. Circumcision is practiced in many religions and traditions.

6. The postpartum client is concerned about mastitis because she experienced it with her last baby. Preventive measures the nurse can teach include which of the following? A) Wearing a tight-fitting bra B) Limiting breastfeedings C) Frequent breastfeedings D) Restricting fluid intake

Answer: C Explanation: A) Although wearing a supportive bra is recommended, a tight-fitting bra would tend to suppress lactation. B) The woman should continue to breastfeed; in fact, regular drainage of both breasts actually helps by preventing milk stasis and abscess formation. C) Treatment and prevention of mastitis includes frequent and complete emptying of the breasts. D) Supportive measures include increased fluid intake (at least 2 to 2.5 L/day

8. Which relief measure would be most appropriate for a postpartum client with superficial thrombophlebitis? A) Urge ambulation B) Apply ice to the leg C) Elevate the affected limb D) Massage her

Answer: C Explanation: A) Ambulation would increase discomfort. B) Applying ice to the leg is contraindicated in thrombophlebitis treatment. Treatment includes application of local heat. C) Treatment for superficial thrombophlebitis involves application of local heat, elevation of the affected limb, and analgesic agents. D) Massaging the calf is contraindicated because it can cause a breakup of a clot and put the client at risk for a pulmonary embolus. Pulmonary embolism occurs when a thrombus from a lower extremity or the pelvis lodges in the pulmonary vascular bed and restricts circulation to the corresponding area of the lung vasculature.

21)A new grandfather is marveling over his 12-hour-old newborn grandson. Which statement indicates that the grandfather needs additional education? A) "I can't believe he can already digest fats, carbohydrates, and proteins." B) "It is amazing that his whole digestive tract can move things along at birth." C) "Incredibly, his stomach capacity was already a cupful when he was born." D) "He will lose some weight but then miraculously regain it by about 10 days

Answer: C Explanation: A) At birth, neonates can digest fats, simple carbohydrates, and proteins. B) The stomach empties intermittently, starting within a few minutes of the beginning of a feeding and ending between 2 and 4 hours after feeding. C) The newborn's stomach has a capacity of 50 to 60 mL. D) Following birth, caloric intake is often insufficient for weight gain until the newborn is 5 to 10 days old. During this time there may be a weight loss of 5% to 10% in term newborns.

12)On the first postpartum day, the nurse teaches the client about breastfeeding. Two hours later, the mother seems to remember very little of the teaching. The nurse understands this memory lapse to be related to which of the following? A) The taking-hold phase B) Postpartum hemorrhage C) The taking-in period D) Epidural anesthesia

Answer: C Explanation: A) By the second or third day after birth, the new mother may be observed to be ready to resume control of her body, her mothering, and her life in general. This is called the taking-hold phase. B) Postpartum hemorrhage is a serious complication and needs medical intervention. A low or decreasing blood pressure may reflect hypovolemia secondary to hemorrhage. C) Soon after birth during the taking-in period, the woman tends to be passive and somewhat dependent. She follows suggestions, hesitates about making decisions, and is still rather preoccupied with her needs. D) Epidural anesthesia is a pharmacological approach to pain control. There may be an increased incidence of headache if the woman had spinal or epidural anesthesia.

2) A laboring mother has recurrent late decelerations. At birth, the infant has a heart rate of 100, is not breathing, and is limp and bluish in color. What nursing action is best? A) Begin chest compressions. B) Begin direct tracheal suctioning. C) Begin bag-and-mask ventilation. D) Obtain a blood pressure reading.

Answer: C Explanation: A) Chest compressions are not initiated until the heart rate is less than 60 and respirations have been established. B) Direct tracheal suctioning would be appropriate if there were meconium-stained fluid. There is no information about the amniotic fluid in the question. C) Most newborns can be effectively resuscitated by bag-and-mask ventilation. D) Blood pressure is insignificant during resuscitation efforts. This infant needs respirations established

10)The nurse is caring for an infant who was delivered in a car on the way to the hospital and who has developed cold stress. Which finding requires immediate intervention? A) Increased skin temperature and respirations B) Blood glucose level of 45 C) Room-temperature IV running D) Positioned under radiant warmer

Answer: C Explanation: A) Decreased skin temperature and decreased respirations are signs and symptoms of cold stress. B) A blood glucose level of 45 is an adequate blood sugar in a neonate. A level lower than 40 indicates the infant is hypoglycemic. C) IV fluids should be warmed prior to administration and the newborn can be wrapped in a chemically activated warming mattress immediately following birth to decrease the postnatal fall in temperature that normally occurs. D) Radiant warmers are used to gradually increase the neonate's temperature.

27)The postpartum client is about to go home. The nurse includes which subject in the teaching plan? A) Replacement of fluids B) Striae C) Diastasis of the recti muscles D) REEDA scale

Answer: C Explanation: A) Frequently, the woman is quite thirsty following birth and will drink large amounts of fluid. Drinking fluids helps replace fluid lost during labor, in the urine, and through perspiration. This is not something that is taught to the patient at discharge. B) Striae (stretch marks) are a normal skin change in the pregnant woman. C) Diastasis recti abdominis and abdominal muscle tone can be improved with exercise. These exercises can be taught during the postpartum period prior to discharge. D) To evaluate the state of healing, the nurse inspects the Cesarean wound for redness, edema, ecchymosis, discharge, and approximation (REEDA scale). This is not something that is taught to the patient at discharge.

4) The nurse is using the New Ballard Score to assess the gestational age of a newborn delivered 4 hours ago. The infant's gestational age is 33 weeks based on early ultrasound and last menstrual period. The nurse expects the infant to exhibit which of the following? A) Full sole creases, nails extending beyond the fingertips, scarf sign showing the elbow beyond the midline B) Testes located in the upper scrotum, rugae covering the scrotum, vernix covering the entire body C) Ear cartilage folded over, lanugo present over much of the body, slow recoil time D) 1 cm breast bud, peeling skin and veins not visible, rapid recoil of legs and arms to extension

Answer: C Explanation: A) Full sole creases and nails beyond the fingertips are seen in term infants; a scarf sign beyond the midline is characteristic of a preterm infant. B) Testes in the upper scrotum and rugae-covered scrotum are seen in term infants. Vernix covering the body is an indication of a preterm infant. C) Ear cartilage folded over, lanugo present over much of the body, and slow recoil time are all characteristics of a preterm infant. D) 1 cm breast bud, peeling skin, the presence of adipose tissue so that veins are not visible, and rapid recoil of the legs and arms are all indications of term or post-term infants.

19)Which of the following is the primary carbohydrate in the breastfeeding newborn? A) Glucose B) Fructose C) Lactose D) Maltose

Answer: C Explanation: A) Glucose is not the primary carbohydrate in the breastfeeding newborn. B) Fructose is not the primary carbohydrate in the breastfeeding newborn. C) Lactose is the primary carbohydrate in the breastfeeding newborn and is generally easily digested and well absorbed. D) Newborns have trouble digesting starches (changing more complex carbohydrates into maltose), so they should not eat them until after the first 6 months of life.

4) A nurse explains to new parents that their newborn has developed respiratory distress syndrome (RDS). Which of the following signs and symptoms would not be characteristic of RDS? A) Grunting respirations B) Nasal flaring C) Respiratory rate of 40 during sleep D) Chest retractions

Answer: C Explanation: A) Grunting with respirations is a characteristic of RDS. B) Nasal flaring is a characteristic of RDS. C) A respiratory rate of 40 during sleep is normal. D) Significant chest retractions are characteristic of RDS.

3) The nurse is caring for an infant born at 37 weeks that weighs 1750 g (3 pounds 10 ounces). The head circumference and length are in the 25th percentile. What statement would the nurse expect to find in the chart? A) Preterm appropriate for gestational age, symmetrical IUGR B) Term small for gestational age, symmetrical IUGR C) Preterm small for gestational age, asymmetrical IUGR D) Preterm appropriate for gestational age, asymmetrical IUGR

Answer: C Explanation: A) Head circumference and length between the 10th and 90th percentiles indicate asymmetrical IUGR. B) Head circumference and length between the 10th and 90th percentiles indicate asymmetrical IUGR. C) The infant is preterm at 37 weeks. Because the weight is below the 10th percentile, the infant is small for gestational age. Head circumference and length between the 10th and 90th percentiles indicate asymmetrical IUGR. D) The infant is preterm at 37 weeks. Because the weight is below the 10th percentile, the infant is considered small for gestational age.

29)Specific cellular immunity is mediated by T lymphocytes, which enhance the efficiency of the phagocytic response. What do cytotoxic activated T cells do? A) Enable T or B cells to respond to antigens B) Repress responses to specific B or T lymphocytes to antigens C) Kill foreign or virus-infected cells D) Remove pathogens and cell debris

Answer: C Explanation: A) Helper activated T cells enable T or B cells to respond to antigens. B) Suppressor activated T cells repress responses to specific B or T lymphocytes to antigens. C) Cytotoxic activated T cells kill foreign or virus-infected cells. D) Phagocytosis is a major mechanism to remove pathogens and cell debris

19)The nurse has instructed a new mother on quieting activities for her newborn. The nurse knows that the mother understands when she overhears the mother telling the father to do what? A) Hold the newborn in an upright position. B) Massage the hands and feet. C) Swaddle the newborn in a blanket. D) Make eye contact while talking to the newborn.

Answer: C Explanation: A) Holding the newborn upright is a waking activity. B) Increasing skin contact and gently rubbing hands and feet is a waking activity. C) Swaddling or bundling the baby increases the sense of security and is a quieting activity. D) Talking to the newborn while making eye contact is a waking activity

19)The postpartum nurse is caring for a client who gave birth to full-term twins earlier today. The nurse will know to assess for symptoms of which of the following? A) Increased blood pressure B) Hypoglycemia C) Postpartum hemorrhage D) Postpartum infection

Answer: C Explanation: A) Increased blood pressure would cause vasoconstriction, and is not identified in this client. B) Hypoglycemia would not be a usual assessment for the mother unless she was diabetic, and this is not identified in this client. C) The nurse will assess for postpartum hemorrhage. Overstretching of uterine muscles with conditions such as multiple gestation, polyhydramnios, or a very large baby may set the stage for slower uterine involution. D) Postpartum infection would be assessed through lochia odor. The odor of the lochia is nonoffensive and never foul. If foul odor is present, so is an infection. Infection would not be expected this soon after delivery.

6) Which of the following may lead to the development of disseminated intravascular coagulation (DIC), also called consumption coagulopathy, in the mother? A) Hypertensive disorders B) Abruptio placentae C) Prolonged retention of the dead fetus D) Heritable thrombophilias

Answer:C Explanation: A) Hypertensive disorders may contribute to the incidence of stillbirth. B) Abruptio placentae contributes to the incidence of stillbirth. C) Prolonged retention of the dead fetus may lead to the development of disseminated intravascular coagulation (DIC), also called consumption coagulopathy, in the mother. D) Heritable thrombophilias may contribute to the incidence of stillbirth

21)During discharge planning for a drug-dependent newborn, the nurse explains to the mother how to do which of the following? A) Place the newborn in a prone position. B) Limit feedings to three a day to decrease diarrhea. C) Place the infant supine and operate a home apnea-monitoring system. D) Wean the newborn off the pacifier.

Answer: C Explanation: A) Infants with neonatal abstinence syndrome are at a significantly higher risk for sudden infant death syndrome (SIDS) when the mother used heroin, cocaine, or opiates. The infant should sleep in a supine position, and home apnea monitoring should be implemented. B) Small, frequent feedings are recommended. C) Infants with neonatal abstinence syndrome are at a significantly higher risk for sudden infant death syndrome (SIDS) when the mother used heroin, cocaine, or opiates. The infant should sleep in a supine position, and home apnea monitoring should be implemented. D) A pacifier may be offered to provide nonnutritive sucking.

15)The nurse is teaching a class on infant care to new parents. Which statement by a parent indicates that additional teaching is needed? A) "The white spots on my baby's nose are called milia, and are harmless." B) "The whitish cheeselike substance in the creases is vernix, and will be absorbed." C) "The red spots with a white center on my baby are abnormal acne." D) "Jaundice is a yellowish discoloration of skin that if noticed on the 1st day of life should be reported to the physician."

Answer: C Explanation: A) Milia are exposed sebaceous glands, and appear as white spots, often across the nose. B) Disappearance of the protective vernix caseosa promotes skin desquamation. C) Red spots with white or yellow centers are erythema toxicum. The peak incidence is at 24 to 48 hours of life. The condition rarely presents at birth or after 5 days of life. The cause is unknown, and no treatment is necessary. D) Jaundice is a yellowish discoloration of skin and mucous membranes. Any jaundice noted before 24 hours of age should be reported to the physician or nurse practitioner.

32)The community nurse is working with a client whose only child is 8 months old. Which statement does the nurse expect the mother to make? A) "I have a lot more time to myself than I thought I would have." B) "My confidence level in my parenting is higher than I anticipated." C) "I am constantly tired. I feel like I could sleep for a week." D) "My baby likes everyone, and never fusses when she's held by a stranger."

Answer: C Explanation: A) Most new mothers have difficulty finding time for themselves. B) Feelings of incompetence at parenting are also common. C) Physical fatigue often affects adjustments and functions of the new mother. The nurse can also provide information about the fatigue that a new mother experiences, strategies to promote rest and sleep at home, and the impact fatigue can have on a woman's emotions and sense of control. D) At 8 months, some infants develop stranger anxiety, and they will cry when held by anyone other than the parents.

16)During an assessment of a 12-hour-old newborn, the nurse notices pale pink spots on the nape of the neck. The nurse documents this finding as which of the following? A) Nevus vasculosus B) Nevus flammeus C) Telangiectatic nevi D) A Mongolian

Answer: C Explanation: A) Nevus vasculosus is a strawberry hemangioma. B) Nevus flammeus is a port-wine stain hemangioma. C) Telangiectatic nevi (stork bites) appear as pale pink or red spots and are frequently found on the eyelids, nose, lower occipital bone, and nape of the neck. D) Congenital dermal melanocytosis (Mongolian blue spots) are macular areas of bluish black or gray-blue pigmentation commonly found on the dorsal area and the buttocks but may be anywhere on the body.

25)The nurse attempts to elicit the Moro reflex on a newborn, and assesses movement of the right arm only. Based on this finding, the nurse immediately assesses for which of the following? A) Ortolani maneuver B) Palmar grasping reflex C) Clavicle D) Tonic neck reflex

Answer: C Explanation: A) Ortolani maneuver is an assessment technique that rules out the possibility of hip dislocation or hip instability. B) Palmar grasping reflex is elicited by stimulating the newborn's palm with a finger or object; the newborn grasps and holds the object or finger firmly enough to be lifted momentarily from the crib. C) When the Moro reflex is elicited, the newborn straightens arms and hands outward while the knees flex. Slowly the arms return to the chest, as in an embrace. If this response is not elicited, the nurse assesses the clavicle for a possible fracture. D) Tonic neck reflex (fencer position) is elicited when the newborn is supine and the head is turned to one side. In response, the extremities on the same side straighten, whereas on the opposite side they flex.

19.The client delivered her second child 1 day ago. The client's temperature is 101.4° F, her pulse is 100, and her blood pressure is 110/70. Her lochia is moderate, serosanguinous, and malodorous. She is started on IV antibiotics. The nurse provides education for the client and her partner. Which statement indicates that teaching has been effective? A) "This condition is called parametritis." B) "Gonorrhea is the most common organism that causes this type of infection." C) "My positive Beta-strep culture might have contributed to this problem." D) "If I had walked more yesterday, this probably wouldn't have happened."

Answer: C Explanation: A) Pelvic cellulitis (parametritis) is an infection that has ascended to involve the connective tissue of the broad ligament or, in more severe forms, the connective tissue of all the pelvic structures. B) Gonorrhea is not a common cause of endometritis. C) Clinical findings of endometritis in the initial 24 to 36 hours postpartum tend to be related to group B streptococcus (GBS). D) Walking would prevent deep vein thrombophlebitis, not endometritis.

11)The nurse is planning care for a newborn. Which nursing intervention would best protect the newborn from the most common form of heat loss? A) Placing the newborn away from air currents B) Pre-warming the examination table C) Drying the newborn thoroughly D) Removing wet linens from the isolette

Answer: C Explanation: A) Placing the newborn away from air currents reduces heat loss by convection, which is not the most common form of heat loss. B) Pre-warming the examination table reduces heat loss by conduction, which is not the most common form of heat loss. C) The most common form of heat loss is evaporation. The newborn is particularly prone to heat loss by evaporation immediately after birth (when the baby is wet with amniotic fluid) and during baths; thus drying the newborn is critical. D) Removing wet linens from the isolette that are not in direct contact with the newborn reduces heat loss by radiation, which is not the most common form of heat loss.

30)A nurse is caring for several postpartum clients. Which client is demonstrating a problem attaching to her newborn? A) The client who is discussing how the baby looks like her father B) The client who is singing softly to her baby C) The client who continues to touch her baby with only her fingertips D) The client who picks her baby up when the baby cries

Answer: C Explanation: A) Pointing out family traits or characteristics in the newborn is a positive sign of developing early attachment. B) Speaking or singing to the baby frequently and affectionately is a positive sign of developing early attachment. C) In a progression of touching activities, the mother proceeds from fingertip exploration of the newborn's extremities toward palmar contact with larger body areas and finally to enfolding the infant with the whole hand and arms. If the client continues to touch with only her fingertips, she might not be developing adequate early attachment. D) Being sensitive to the newborn's needs is a positive sign of developing early attachment.

27)The parents of a preterm newborn wish to visit their baby in the NICU. A statement by the nurse that would not support the parents as they visit their newborn is which of the following? A) "Your newborn likes to be touched." B) "Stroking the newborn will help with stimulation." C) "Visits must be scheduled between feedings." D) "Your baby loves her pink blanket."

Answer: C Explanation: A) Statements that encourage the parents to touch the newborn will help them bond with their child. B) Statements that encourage the parents to stroke the newborn will help them bond with their child and provide stimulation. C) The nurse should always encourage parents to visit and get to know their newborn, even in the NICU. Nurses foster the development of a safe, trusting environment by viewing the parents as essential caregivers, not as visitors or nuisances in the unit. D) Comments that personalize the baby will tell the parents their baby is unique and special.

12)The nurse is planning an educational presentation on hyperbilirubinemia for nursery nurses. Which statement is most important to include in the presentation? A) Conjugated bilirubin is eliminated in the conjugated state. B) Unconjugated bilirubin is neurotoxic, and cannot cross the placenta. C) Total bilirubin is the sum of the direct and indirect levels. D) Hyperbilirubinemia is a decreased total serum bilirubin level.

Answer: C Explanation: A) The (direct) conjugated bilirubin progresses down the intestines, where bacteria transform it into urobilinogen (urine bilirubin). Even after the bilirubin has been conjugated and bound, it can be changed back to unconjugated bilirubin via the enterohepatic circulation. B) Fetal unconjugated bilirubin crosses the placenta to be excreted, so the fetus does not need to conjugate bilirubin. C) Total serum bilirubin is the sum of conjugated (direct) and unconjugated (indirect) bilirubin. D) Hyperbilirubinemia is an elevated total serum bilirubin level.

24)The nurse is instructing a new mother on circumcision care with a Plastibell. The nurse knows the mother understands when she states that the Plastibell should fall off within how long? A) 2 days B) 10 days C) 8 days D) 14 days

Answer: C Explanation: A) The Plastibell does not fall off in 2 days. B) The Plastibell should fall off before 10 days. C) The Plastibell should fall off within 8 days. If it remains on after 8 days, the parents should consult with the newborn's physician. D) The Plastibell should fall off before 14 days.

2) A 2-day-old newborn is asleep, and the nurse assesses the apical pulse to be 88 beats/min. What would be the most appropriate nursing action based on this assessment finding? A) Call the physician. B) Administer oxygen. C) Document the finding. D) Place the newborn under the radiant warmer.

Answer: C Explanation: A) The apical pulse rate is within normal range. There is no need to call the physician. B) There is no need to administer oxygen at this time. C) An apical pulse rate of 88 beats/min is within the normal range of a sleeping full- term newborn. The average resting heart rate in the first week of life is 110 to 160 beats/min in a healthy full-term newborn but may vary significantly during deep sleep or active awake states. In full-term newborns, the heart rate may drop to a low of 80 to 100 beats/min during deep sleep. D) There is no need to place the infant in a radiant warmer.

1. The charge nurse is assessing several postpartum clients. Which client has the greatest risk for postpartum hemorrhage? A) The client who was overdue and delivered vaginally B) The client who delivered by scheduled cesarean delivery C) The client who had oxytocin augmentation of labor D) The client who delivered vaginally at 36 weeks

Answer: C Explanation: A) The client who was overdue and delivered vaginally has a lower risk for postpartum hemorrhage than would another client. B) The client who delivered by scheduled cesarean delivery has a lower risk for postpartum hemorrhage than would another client. C) Uterine atony is a cause of postpartal hemorrhage. A contributing factor to uterine atony is oxytocin augmentation of labor. D) The client who delivered vaginally at 36 weeks has a lower risk for postpartum hemorrhage than would another client.

25)New parents decide not to have their newborn circumcised. What should the nurse teach regarding care for the uncircumcised infant? A) The foreskin will be retractable at 2 months. B) Retract the foreskin and clean thoroughly. C) Avoid retracting the foreskin. D) Use soap and Betadine to cleanse the penis daily.

Answer: C Explanation: A) The foreskin is not fully retractable at 2 months. B) The foreskin is not fully retractable in a newborn, and should not be forced back over the penis. C) The foreskin will retract normally over time, and may take 3 to 5 years. D) If retraction has occurred, daily gentle washing of the glans with soap and water is sufficient to maintain adequate cleanliness.

21)The nurse is working with a student nurse during assessment of a 2-hour-old newborn. Which action indicates that the student nurse understands neonatal assessment? A) The student nurse listens to bowel sounds, then assesses the head for skull consistency and size and tension of fontanelles. B) The student nurse checks for Ortolani's sign, then palpates the femoral pulse, then assesses respiratory rate. C) The student nurse determines skin color, then describes the shape of the chest and looks at structures and flexion of the feet. D) The student nurse counts the number of cord vessels, then assesses genitals, then sclera color and eyelids.

Answer: C Explanation: A) The head should be assessed before the bowel sounds. B) The respiratory rate should be assessed first, when the infant is at rest and undisturbed. C) Neonatal assessment proceeds in a head-to-toe fashion. D) The sclera and eye assessment should be done prior to assessing genitals

13)A telephone triage nurse gets a call from a postpartum client who is concerned about jaundice. The client's newborn is 37 hours old. What data point should the nurse gather first? A) Stool characteristics B) Fluid intake C) Skin color D) Bilirubin level

Answer: C Explanation: A) The stool characteristic of green coloration indicates excretion of bilirubin. B) Breastfeeding is implicated in jaundice in some newborns. Breast milk jaundice occurs in approximately 2% to 4% of term infants with an onset of 4 to 7 days of life. C) Jaundice (icterus) is the yellowish coloration of the skin and sclera caused by the presence of bilirubin in elevated concentrations. Inspection of the skin would be the first step in assessing for jaundice. D) Bilirubin is primarily the metabolic end product of erythrocyte (RBC) breakdown. Conjugation, or the changing of bilirubin into an excretable form, is the conversion of the yellow lipid-soluble pigment (unconjugated, indirect) into water- soluble pigment (excretable, direct).

9) The nurse is providing discharge teaching to a woman who delivered her first child 2 days ago. The nurse understands that additional information is needed if the client makes which statement? A) "I should expect a lighter flow next week." B) "The flow will increase if I am too active." C) "My bleeding will remain red for about a month." D) "I will be able to use a pantiliner in a day or two."

Answer: C Explanation: A) The uterus rids itself of the debris remaining after birth through a discharge called lochia. Lochia serosa is a pinkish color. It follows from about the 3rd to the 10th day toward a lighter amount of flow and a lighter color of discharge. When the lochia stops, the cervix is considered closed. B) The amount of lochia may be increased by exertion or breastfeeding. C) Lochia rubra is dark red in color. It is present for the first 2 to 3 days postpartum. Lochia serosa is a pinkish color and it follows from about the 3rd to the 10th day. D) By day three, the lochia shed is a lighter amount of flow and a lighter color of discharge. A pantiliner would be appropriate for use at this point.

28)The nurse is completing a newborn care class. The nurse knows that teaching has been effective if a new parent states which of the following? A) "My baby might open her arms wide and pull her legs up to her tummy if she is passing gas." B) "When I hold my baby upright with one of his feet on the floor, his feet will automatically remain still." C) "When I put my finger in the palm of my daughter's hand, she will curl her fingers and hold on." D) "I can get my baby to turn his head toward the right if I lift his right arm over his head."

Answer: C Explanation: A) This is the Moro or startle reflex, and occurs when the infant is startled by a sudden movement or a loud noise. B) Stepping reflex happens when a newborn who is held upright with one foot touching a flat surface puts one foot in front of the other and "walks." C) This is the Palmar grasp reflex and is elicited by stimulating the newborn's palm with a finger or object. D) This is the tonic neck reflex and is elicited when the newborn is supine and the head is turned to one side. In response, the extremities on the same side straighten, whereas on the opposite side they flex.

11)Which statement by a new mother 1 week postpartum indicates maternal role attainment? A) "I don't think I'll ever know what I'm doing." B) "This baby feels like a real stranger to me." C) "It works better for me to undress the baby and to nurse in the chair rather than the bed." D) "My sister took to mothering in no time. Why can't I?"

Answer: C Explanation: A) This statement indicates that the mother has not yet achieved a level of comfort or confidence in her new role as a mother. B) This statement indicates that "binding in" to the newborn has not yet occurred. C) This statement indicates a stage of maternal role attainment in which the new mother feels comfortable enough to make her own decisions about parenting. D) This statement indicates that the mother has not yet achieved a level of comfort or confidence in her new role.

18)The nurse is planning care for a preterm newborn. Which nursing diagnosis has the highest priority? A) Tissue Integrity, Impaired B) Infection, Risk for C) Gas Exchange,Impaired D) Family Processes, Dysfunctional

Answer: C Explanation: A) Tissue Integrity, Impaired is related to fragile capillary network in the germinal matrix, but is not the highest priority. B) Infection, Risk for isrelated to lack of passive immunity and immature immune defenses due to preterm birth, but is not the highest priority. C) Gas Exchange, Impaired is related to immature pulmonary vasculature and inadequate surfactant production, and has the highest priority. D) Family Processes, Dysfunctionalis related to anger or guilt at having given birth to a premature baby and is a psychosocial need, and is therefore a lower priority than are physiologic needs.

20)The nurse is observing a new graduate perform a postpartum assessment. Which action requires intervention by the nurse? A) Asking the client to void and donning clean gloves B) Listening to bowel sounds and then asking when her last bowel movement occurred C) Offering the patient pre-medication 2 hours before the assessment D) Completing the assessment and explaining the results to the client

Answer: C Explanation: A) Voiding prior to the assessment helps ensure comfort; clean gloves prevent exposure to body fluids. B) It is appropriate to ask about a body part when assessing that part. C) The patient should be offered premedication 30-45 minutes before assessing the fundus, especially if the patient has had a cesarean section. D) The nurse should provide an explanation of the purposes of regular assessment to the woman.

17)The nurse teaches the parents of an infant who was recently circumcised to observe for bleeding. What should the parents be taught to do if bleeding does occur? A) Wrap the diaper tightly. B) Clean with warm water with each diaper change. C) Apply gentle pressure to the site with gauze. D) Apply a new petroleum ointment gauze dressing.

Answer: C Explanation: A) When diapering, ensure that the diaper is neither too loose, which can cause rubbing with movement, nor too tight, which can cause pain. B) Cleaning the newborn with warm water with each diaper change is part of the care plan, but it does not prevent bleeding. C) If bleeding does occur, apply light pressure with a sterile gauze pad to stop the bleeding within a short time. If this is not effective, contact the physician immediately or take the newborn to the healthcare provider. D) Continued application of a petroleum ointment can help protect the granulation tissue that forms as the glans heals, but does not stop any bleeding

10)As the couple and their families begin to confront the pain of their loss, many normal manifestations of grief may be present. Which of the following would indicate an emotional response to the loss? A) Lack of meaning or direction B) Preoccupation C) Flat affect D) Dreams of the deceased

Answer:C Explanation: A) Lack of meaning or direction would be a spiritual response to loss. B) Preoccupation would be a cognitive response to loss. C) Flat affect would be an emotional response to loss. D) Dreams of the deceased would be a behavioral response to loss.

33)The nurse is preparing a class for mothers who have just recently delivered and their partners. One topic of the class is infant attachment. Which statement by a participant indicates an understanding of this concept? Note: Credit will be given only if all correct choices and no incorrect choices are selected. Select all that apply. A) "We should avoid holding the baby too much." B) "Looking directly into the baby's eyes might frighten him." C) "Talking to the baby is good because he'll recognize our voices." D) "Holding the baby so we have direct face-to-face contact is good." E) "We should only touch the baby with our fingertips for the first month

Answer: C, D Explanation: A) Attachment behaviors include holding the baby in the en face position. B) Attachment behaviors include making eye contact. C) Attachment behaviors include cuddling, soothing, and calling the baby by name. D) Attachment behaviors include holding the baby in the en face position. E) In a progression of touching activities, the mother proceeds from fingertip exploration of the newborn's extremities toward palmar contact with larger body areas and finally to enfolding the infant with the whole hand and arms. The time taken to accomplish these steps varies from minutes to days.

17)Clinical risk factors for severe hyperbilirubinemia include which of the following? Note: Credit will be given only if all correct choices and no incorrect choices are selected. Select all that apply. A) African American ethnicity B) Female gender C) Cephalohematoma D) Bruising E) Assisted delivery with vacuum or forceps

Answer: C, D, E Explanation: A) A clinical risk factor for severe hyperbilirubinemia includes Asian ethnicity. B) A clinical risk factor for severe hyperbilirubinemia includes male gender. C) A clinical risk factor for severe hyperbilirubinemia includes cephalohematoma. D) A clinical risk factor for severe hyperbilirubinemia includes bruising. E) A clinical risk factor for severe hyperbilirubinemia includes assisted delivery with vacuum or forceps.

14)The nurse is working with parents who have just experienced the birth of their first child at 34 weeks. Which statement(s) by the parents indicate that additional teaching is needed? Note: Credit will be given only if all correct choices and no incorrect choices are selected. Select all that apply. A) "Our baby will be in an incubator to keep him warm." B) "Breathing might be harder for our baby because he is early." C) "The growth of our baby will be faster than if he were term." D) "Tube feedings will be required because his stomach is small." E) "Because he came early, he will not produce urine for 2 days.

Answer: C, D, E Explanation: A) Preterm infants have little subcutaneous fat, and have difficulty maintaining their body temperature. An incubator or warmer is used to keep the baby warm. B) Surfactant production might not be complete at 34 weeks, which leads to respiratory distress syndrome. The infant may become hypoxic, pulmonary blood flow may be inefficient, and the preterm newborn's available energy is depleted. C) Preterm infants grow more slowly than do term infants because of difficulty in meeting high caloric and fluid needs for growth due to small gastric capacity. D) Although tube feedings might be required, it would be because preterm babies have a marked danger of aspiration and its associated complications due to the infant's poorly developed gag reflex, incompetent esophageal cardiac sphincter, and inadequate suck/swallow/breathe reflex. E) Although preterm babies have diminished kidney function due to incomplete development of the glomeruli, they can produce urine. Preterm infants usually have some urine output during the first 24 hours of life.

20)The nurse is assessing the newborn for symptoms of anemia. If the blood loss is acute, the baby may exhibit which of the following signs of shock? Note: Credit will be given only if all correct choices and no incorrect choices are selected. Select all that apply. A) Increased pulse B) High blood pressure C) Tachycardia D) Bradycardia E) Capillary filling time greater than 3 seconds

Answer: C, E Explanation: A) Decreased pulse would be a sign of shock. B) Low blood pressure would be a sign of shock. C) Tachycardia would be a sign of shock. D) Tachycardia, not bradycardia, would be a sign of shock. E) Capillary filling time greater than 3 seconds would be a sign of shock

22)During a postpartum examination of a client who delivered an 8-pound newborn 6 hours ago, the following assessment findings are noted: fundus firm and at the umbilicus, and moderate lochia rubra with a steady trickle of blood from the vagina. What is the assessment finding that would necessitate follow-up? A) Firm fundus B) Fundus at the umbilical level C) Moderate lochia rubra D) Steady trickle of blood

Answer: D Explanation: A) A firm fundus is desirable, and considered normal. B) Six hours after birth, the fundus at the umbilicus would not be a concern. C) Lochia rubra would be considered normal. D) The continuous seepage of blood is more consistent with cervical or vaginal lacerations. Lacerations should be suspected if the uterus is firm and of expected size, and no clots can be expressed. This finding would indicate a follow-up

7) The nurse assesses the newborn and notes the following behaviors: nasal flaring, facial grimacing, and excessive mucus. What is the nurse most concerned about? A) Neonatal jaundice B) Neonatal hypothermia C) Neonatal hyperthermia D) Respiratory distress

Answer: D Explanation: A) A high bilirubin level would be an indication of jaundice. B) Temperature instability would indicate either hyperthermia or hypothermia. C) A high temperature would indicate hyperthermia. D) Nasal flaring and facial grimacing are signs of respiratory distress.

30.The postpartum client who is being discharged from the hospital experienced severe postpartum depression after her last birth. What should the nurse include in the plan of follow-up care for this client? A) One visit from a home care nurse, to take place in 2 days B) Two visits from a public health nurse over the next month C) An appointment with a mental health counselor D) Follow-up with the obstetrician in 6 weeks

Answer: D Explanation: A) A home visit in 2 days will be helpful to assess feeding, but is too early to detect signs of postpartum depression. B) Two home visits in a month are too sporadic to accurately detect postpartum depression. C) Postpartum depression has a high recurrence rate. Women with a history of postpartum psychosis or depression, or other risk factors, may benefit from a referral to a mental health professional for counseling during pregnancy or postpartum. D) Following up with the obstetrician in 6 weeks is too long a wait

24)The nurse is performing a postpartum assessment on a newly delivered client. When checking the fundus, there is a gush of blood. The client asks why that is happening. What is the nurse's best response? A) "We see this from time to time. It's not a big deal." B) "The gush is an indication that your fundus isn't contracting." C) "Don't worry. I'll make sure everything is fine." D) "Blood pooled in the vagina while you were in bed."

Answer: D Explanation: A) Although a gush of blood during fundus assessment is fairly common, this response is not therapeutic, because it does not answer the client's question. B) The fundus might be contracting well. The gush is from pooled lochia in the vagina. C) This response is not therapeutic because it focuses on the nurse and has a "don't worry" aspect that most clients find demeaning. D) A gush of blood when a fundal massage is undertaken may occur because of normal pooling of blood in vagina when the woman lies down to rest or sleep

30)The nurse is working with an adolescent mother who tells the nurse, "I'm really scared that I won't take care of my baby correctly. My mother says I'll probably hurt the baby because I'm too young to be a mother." What is the best response by the nurse? A) "You are very young, and parenting will be a challenge for you." B) "Your mother was probably right. Be very careful with your baby." C) "Mothers have instincts that kick in when they get their babies home." D) "We can give the baby a bath together. I'll help you learn how to do it."

Answer: D Explanation: A) Although this statement is true, it does not teach the client anything, or increase her confidence in being able to care for her infant. B) This statement is very judgmental, and does not teach the client anything, or increase her confidence in being able to care for her infant. C) Maternal instincts might indeed exist, but this client has expressed a specific fear about being a safe mother. It is best to work with her to teach her skills and increase her confidence. D) This response is best because bathing the newborn offers an excellent opportunity for teaching and welcoming parent involvement in the care of their baby.

5. A client had a cesarean birth 3 days ago. She has tenderness, localized heat, and redness of the left leg. She is afebrile. As a result of these symptoms, what would the nurse anticipate would be the next course of action? A) That the client would be encouraged to ambulate freely B) That the client would be given aspirin 650 mg by mouth C) That the client would be given Methergine IM D) That the client would be placed on bed rest

Answer: D Explanation: A) Ambulation would increase the inflammation. B) Aspirin 650 mg by mouth has anticoagulant properties, but usually is not necessary unless complications occur. C) Methergine is given only for postpartum hemorrhage, and would only cause vasoconstriction of an already inflamed vessel. D) These symptoms indicate the presence of superficial thrombophlebitis. The treatment involves bed rest, elevation of the affected limb, analgesics, and use of elastic support hose.

30)The nurse is analyzing assessment findings on four newborns. Which finding might suggest a congenital heart defect? A) Apical heart rate of 140 beats per minute B) Respiratory rate of 40 C) Temperature of 36.5°C D) Visible, blue discoloration of the skin

Answer: D Explanation: A) An apical heart rate of 140 is a normal assessment finding for newborns. B) A respiratory rate of 40 is a normal assessment finding for newborns. C) Temperature of 36.5°C is a normal assessment finding for newborns. D) Central cyanosis is defined as a visible, blue discoloration of the skin caused by decreased oxygen saturation levels and is a common manifestation of a cardiac defect.

6) A 7 pound 14 ounce girl was born to an insulin-dependent typeIIdiabetic mother 2 hours ago. The infant's blood sugar is 47 mg/dL. What is the best nursing action? A) To recheck the blood sugar in 6 hours B) To begin anIV of 10% dextrose C) To feed the baby 1 ounce of formula D) To document the findings in the chart

Answer: D Explanation: A) Blood glucose determinations should be performed on blood by heel stick hourly during the first 4 hours after birth, and subsequently at 4-hour intervals. B) A blood sugar reading of 47 mg/dL is considered normal for a neonate. NoIV is needed. C) Feeding would be appropriate if the infant's blood sugar was below 45 mg/dL, but this infant's reading is 47. D) A blood sugar level of 47 mg/dL is a normal finding; documentation is an appropriate action.

29.The client delivered her second child yesterday, and is preparing to be discharged. She expresses concern to the nurse because she developed an upper urinary tract infection (UTI) after the birth of her first child. Which statement indicates that the client needs additional teaching about this issue? A) "If I start to have burning with urination, I need to call the doctor." B) "Drinking 8 glasses of water each day will help prevent another UTI." C) "I will remember to wipe from front to back after I move my bowels." D) "Voiding 2 or 3 times per day will help prevent a recurrence."

Answer: D Explanation: A) Burning with urination is a common symptom of a UTI. B) Drinking 8 to 10 8-oz glasses of water daily will help to prevent the development of a UTI. C) Wiping from front to back after bowel movements will help to prevent the development of a UTI. D) Voiding only 2 or 3 times per day is not sufficient to prevent recurrence of a urinary tract infection (UTI). The woman needs to empty her bladder whenever she feels the urge to void, or at least every 2 to 4 hours while awake.

31)The parents of a newborn comment to the nurse that their infant seems to enjoy being held, and that holding the baby helps him calm down after crying. They ask the nurse why this happens. After explaining newborn behavior, the nurse assesses the parents' learning. Which statement indicates that teaching was effective? A) "Some babies are easier to deal with than others." B) "We are lucky to have a baby with a calm disposition." C) "Our baby spends more time in the active alert phase." D) "Cuddliness is a social behavior that some babies have."

Answer: D Explanation: A) Calling the baby easy or difficult is a judgment that does not explain the baby's behavior. B) Describing the baby as calm is a judgment that does not explain the baby's behavior. C) The active alert phase of the sleep-awake cycle is characterized by motor activity. It does not explain the baby's behavior D) According to Brazelton Neonatal Behavioral Assessment Scale, cuddliness can be an indicator of personality.

7) The client at 37 weeks' gestation calls the clinic nurse to report that neither she nor her partner has felt fetal movement for the past 48 hours. The nurse anticipates that the physician will order which test to assess fetal viability? A) Ultrasound B) Serum progesterone levels C) Computed tomography (CT) scan D) Contraction stress test

Answer:A Explanation: A) Diagnosis of intrauterine fetal death (IUFD) is confirmed by visualization of the fetal heart with absence of heart action on ultrasound. B) Serum estriol levels drop during fetal demise, but progesterone levels are not measured. C) A computed tomography (CT) scan is not indicated. D) Contraction stress tests are used to determine whether a fetus will tolerate the stresses of labor

27.A postpartum client with endometritis is being discharged home on antibiotic therapy. The new mother plans to breastfeed her baby. What should the nurse's discharge instruction include? A) The client can douche every other day. B) Sexual intercourse can be resumed when the client feels up to it. C) Light housework will provide needed exercise. D) The baby's mouth should be examined for thrush.

Answer: D Explanation: A) Douching is contraindicated for this client. B) Pelvic rest is necessary for this client, and sexual activity should be resumed only when the physician says it is safe. C) The woman with a puerperal infection needs assistance when she is discharged from the hospital. If the family cannot provide this home assistance, a referral to home care services is needed. D) A breastfeeding mother on antibiotics should check her baby's mouth for signs of thrush, which should be reported to the physician.

28)The nurse is discussing parent-infant attachment with a prenatal class. Which statement indicates that teaching was successful? A) "I should avoid looking directly into the baby's eyes to prevent frightening the baby." B) "My baby will be very sleepy immediately after birth and should go to the nursery." C) "Newborns cannot focus their eyes, so it doesn't matter how I hold my new baby." D) "Giving my baby his first bath can really give me a chance to get to know him."

Answer: D Explanation: A) Eye-to-eye contact between parents and their newborn is extremely important during the early hours after birth, when the newborn is in the first period of reactivity. B) Newborns are usually alert and responsive in the first few hours after birth. Interacting with the newborn during this first period of reactivity facilitates parent- infant attachment. C) Newborns can have direct eye contact with human faces, with an optimal range for visual acuity of 7 to 8 inches. Eye contact is an important component of the emerging parent-baby bond. D) Another situation that can facilitate attachment is the interactive bath. While bathing their newborn for the first time, parents attend closely to their baby's behavior and the nurse can observe and point out behaviors.

7) A nurse is caring for a newborn on a ventilator who has respiratory distress syndrome (RDS). The nurse informs the parents that the newborn is improving. Which data support the nurse's assessment? A) Decreased urine output B) Pulmonary vascular resistance increases C) Increased PCO2 D) Increased urination

Answer: D Explanation: A) Increased urination, not decreased urine output, could be an indication that the newborn's condition is improving. B) Pulmonary vascular resistance increases with hypoxia. C) Increased PCO2 results from alveolar hypoventilation. D) In babies with respiratory distress syndrome (RDS) who are on ventilators, increased urination/diuresis may be an early clue that the baby's condition is improving.

4) A 38-week newborn is found to be small for gestational age (SGA). Which nursing intervention should be included in the care of this newborn? A) Monitor for feeding difficulties. B) Assess for facial paralysis. C) Monitor for signs of hyperglycemia. D) Maintain a warm environment.

Answer: D Explanation: A) LGA, not SGA, newborns are more difficult to arouse to a quiet alert state, and can have feeding difficulties. B) LGA, not SGA, newborns often are prone to birth trauma such as facial paralysis, due to cephalopelvic disproportion. C) SGA newborns are more prone to hypoglycemia. D) Hypothermia is a common complication in the SGA newborn; therefore, the newborn's environment must remain warm, to decrease heat loss.

8) To assess the healing of the uterus at the placental site, what does the nurse assess? A) Lab values B) Blood pressure C) Uterine size D) Type, amount, and consistency of lochia

Answer: D Explanation: A) Lab values is an incomplete response because it does not indicate which lab values are referred to. B) Blood pressure varies slightly in normal postpartal women, and would not affect the placental site. C) Uterine size alone is not enough to assess the placental site. D) The type, amount, and consistency of lochia determine the state of healing of the placental site, and a progressive color change from bright red at birth to dark red to pink to white or clear should be observed.

16)Which of the following would be a newborn care procedure that will decrease the probability of high bilirubin levels? A) Monitor urine for amount and characteristics. B) Encourage late feedings to promote intestinal elimination. C) All infants should be routinely monitored for iron intake. D) Maintain the newborn's skin temperature at 36.5°C (97.8°F) or above.

Answer: D Explanation: A) Monitor stool for amount and characteristics. Bilirubin is eliminated in the feces. B) Encourage early feedings to promote intestinal elimination and bacterial colonization, and to provide the caloric intake necessary for hepatic binding proteins to form. C) All infants should be routinely monitored for the development of jaundice. D) Maintain the newborn's skin temperature at 36.5°C (97.8°F) or above; cold stress results in acidosis.

12.Which of the following is a risk factor for urinary retention after childbirth? A) Multiparity B) Precipitous labor C) Unassisted childbirth D) Not sufficiently recovering from the effects of anesthesia

Answer: D Explanation: A) Nulliparity is a risk factor for urinary retention after childbirth. B) Prolonged labor is risk factor for urinary retention after childbirth. C) Instrumental childbirth is a risk factor for urinary retention after childbirth. D) Women who have not sufficiently recovered from the effects of anesthesia and cannot void spontaneously are at risk for urinary retention after childbirth

20)The nurse is caring for a newborn in the special care nursery. The infant has hydrocephalus, and is positioned in a prone position. The nurse is especially careful to cleanse all stool after bowel movements. This care is most appropriate for an infant born with which of the following? A) Omphalocele B) Gastroschisis C) Diaphragmatic hernia D) Myelomeningocele

Answer: D Explanation: A) Omphalocele is a herniation of abdominal contents into the base of the umbilical cord. Hydrocephalus is not associated with an omphalocele. B) Gastroschisis is a full-thickness defect of the abdominal wall that results in the abdominal organs being located on the outside of the body. Hydrocephalus is not associated with a gastroschisis. C) Diaphragmatic hernia is a portion of the intestines in the thoracic cavity due to an abnormal opening in diaphragm, occurring commonly on the left side. Hydrocephalus is not associated with a diaphragmatic hernia. D) Myelomeningocele is a saclike cyst containing meninges, spinal cord, and nerve roots in the thoracic and/or lumbar area. Meticulous cleaning of the buttocks and genitals helps prevent infection. The infant is positioned on his or her abdomen or side and restrained to prevent pressure and trauma to the sac. Hydrocephalus is often present with this condition.

31)The nurse is caring for a client who recently emigrated from a Southeast Asian country. The mother has been resting since the birth, while her sister has changed the diapers and fed the infant. What is the most likely explanation for this behavior? A) The client is not attaching to her infant appropriately. B) The client is not going to be a good mother, and the baby is at risk. C) The client has no mother present to role-model behaviors. D) The client is exhibiting normal behavior for her culture.

Answer: D Explanation: A) Resting since the birth is expected behavior in certain cultures. B) The mother-infant couplet is not at risk. Resting since birth is expected behavior in certain cultures. C) Even though her mother is not present, her sister is role-modeling behaviors. D) Rest, seclusion, and dietary restraint practices in many traditional non-Western cultures (South Asian groups) are designed to assist the woman and her baby during postpartum vulnerable periods.

5) The nurse assesses the postpartum client who has not had a bowel movement by the third postpartum day. Which nursing intervention would be appropriate? A) Encourage the new mother, saying, "It will happen soon." B) Instruct the client to eat a low-fiber diet. C) Decrease fluid intake. D) Obtain an order for a stool softener

Answer: D Explanation: A) Saying "It will happen soon" does not address the client's needs and could increase the chance for constipation. B) Eating a low-fiber diet would not increase bulk or moisture in the stool. C) Decreasing fluid intake would decrease moisture in the fecal material, encouraging constipation. D) Obtaining an order for a stool softener is the correct intervention by the third day. In resisting or delaying the bowel movement, the woman may cause increased constipation and more pain when elimination finally occurs

27)The nurse is analyzing various strategies for teaching new mothers about newborn care. To enhance learning, which teaching method should the nurse implement? A) Select videos on various topics of newborn care. B) Organize a class that includes first-time mothers only. C) Have mothers return in 1 week, when they feel more rested. D) Schedule time for one-to-one teaching in the mother's room.

Answer: D Explanation: A) Selecting videos on various topics of newborn care would not ensure one-to-one teaching. B) Organizing a class that includes first-time mothers only would not ensure one-to- one teaching. C) It is not appropriate or realistic to expect new mothers to return in 1 week. D) One-to-one teaching while the nurse is in the mother's room is shown to be the most effective educational model. Individual instruction is helpful to answer specific questions.

34)The nurse is teaching a group of new parents about newborn behavior. Which statement made by a parent would indicate a need for additional information? A) "Sleep and alert states cycle throughout the day." B) "We can best bond with our child during an alert state." C) "About half of the baby's sleep time is in active sleep." D) "Babies sleep during the night right from birth."

Answer: D Explanation: A) Sleep and alert states are noticeable behaviors in infants, beginning immediately after birth with the first period of alert activity. B) Bonding between infant and parents takes place with interaction during alert states. C) About 45% to 50% of the newborn's total sleep is active sleep, 35% to 45% is quiet sleep, and 10% is transitional between these two periods. D) Over time, the newborn's sleep-wake patterns become diurnal, that is, the newborn sleeps at night and stays awake during the day.

21)The client delivered her first child vaginally 7 hours ago. She has not voided since delivery. She has an IV of lactated Ringer's solution running at 100 mL/hr. Her fundus is firm and to the right of midline. What is the best nursing action? A) To massage the fundus vigorously B) To assess the client's pain level C) To increase the rate of the IV D) To assist the client to the bathroom

Answer: D Explanation: A) The location of the fundus and the time since delivery indicate that the bladder is distended. Emptying the bladder is the top priority. B) To assess the client's pain level is not important at this time. Emptying the bladder is the top priority. C) Emptying the bladder is the top priority. A distended bladder is an indication that hydration is adequate. D) Emptying the bladder is the top priority.

4) The nurse has assessed four newborns' respiratory rates immediately following birth. Which respiratory rate would require further assessment by the nurse? A) 60 breaths per minute B) 70 breaths per minute C) 64 breaths per minute D) 20 breaths per minute

Answer: D Explanation: A) The normal range for respirations of a newborn within 2 hours after birth is 60 to 70 breaths per minute. B) The normal range for respirations of a newborn within 2 hours after birth is 60 to 70 breaths per minute. C) The normal range for respirations of a newborn within 2 hours after birth is 60 to 70 breaths per minute. D) If respirations drop below 20 when the baby is at rest the primary care provider should be notified.

29)The nurse is working with new parents who have recently immigrated to the United States. The nurse is not familiar with the family's cultural background. Which approach is most appropriate when discussing the newborn? A) "You appear to be Muslim. Do you want your son circumcised?" B) "Let me explain newborn care here in the United States." C) "Your baby is a United States citizen. You must be very happy about that." D) "Could you explain your preferences regarding childrearing?"

Answer: D Explanation: A) The nurse should avoid making assumptions about clients based on appearance. B) The nurse should not assume the family doesn't understand the United States healthcare system. C) The nurse should avoid making assumptions regarding family beliefs and values. D) The nurse must be sensitive to the cultural beliefs and values of the family and be aware of cultural variations in newborn care.

33)The nurse is discharging a 15-year-old first-time mother. Which statement should the nurse include in the discharge teaching? A) "Call your pediatrician if the baby's temperature is below 98.6°F axillary." B) "Your baby's stools will change to a greenish color when your milk comes in." C) "You can wipe away any eye drainage that might form." D) "Your infant should wet a diaper at least 6 times per day."

Answer: D Explanation: A) The pediatrician should be called if the temperature is lower than 97.8°F axillary. B) Stool color for a breastfed infant is a yellow gold, soft or mushy stools. C) Eye drainage is abnormal, and should be reported to the baby's provider. D) A minimum of 6 to 10 wet diapers per day indicates adequate fluid intake.

3) The nurse is assessing a newborn at 1 hour of age. Which finding requires an immediate intervention? A) Respiratory rate 60 and irregular in depth and rhythm B) Pulse rate 145, cardiac murmur heard C) Mean blood pressure 55 mmHg D) Pauses in respiration lasting 30 seconds

Answer: D Explanation: A) The respiratory rate is normal. The normal newborn respiratory rate is 30 to 60 breaths per minute. Initial respirations may be largely diaphragmatic, shallow, and irregular in depth and rhythm. B) This pulse rate is normal. The average resting heart rate in the first week of life is 110 to 160 beats/min. Cardiac murmurs are often present in the initial newborn period as transition from fetal to neonatal circulation occurs. C) This is a normal finding in an infant 1 hour old. The average mean blood pressure is 31 to 61 mmHg in full-term resting newborns. D) Pauses in respirations greater than 20 seconds are considered episodes of apnea, and require further intervention.

32)The nurse is instructing the parents of a newborn about car seat safety. Which statement indicates that the parents need additional information? A) "The baby should be in the back seat." B) "Newborns must be in rear-facing car seats." C) "We need instruction on how to use the car seat before installing it." D) "We can bring the baby home from the hospital without a car seat, as it is only a short drive."

Answer: D Explanation: A) The safest spot in any car is the middle of the back seat. The car seat should be positioned to face the rear of the car. B) The safest spot in any car is the middle of the back seat. The car seat should be positioned to face the rear of the car. C) Nurses need to ensure that all parents are knowledgeable about the benefits of child safety seat use and proper installation. Nurses can encourage parents to have their infant safety seats checked by local groups trained specifically for that purpose. D) Newborns must go home from the birthing unit in a car seat adapted to fit newborns.

31)The nurse is planning visits to the homes of new parents and their newborns. Which client should the nurse see first? A) 3-day-old male who received hepatitis B vaccine prior to discharge B) 4-day-old female whose parents are both hearing-impaired C) 5-day-old male with light, sticky, yellow drainage on the circumcision site D) 6-day-old female with greenish discharge from the umbilical cord site

Answer: D Explanation: A) This infant has no indications of unexpected findings. Immunization programs against the hepatitis B virus during the newborn period and infancy are in place in many states. B) This infant is not at risk, but the appointment should be scheduled when the sign language interpreter is available. C) This is normal healing and a light, sticky, yellow drainage may form over the head of the penis. D) Oozing greenish yellow material or reddened areas around the cord is not an expected finding. This family should be seen first because the child is experiencing a complication.

29)The nurse is beginning the postpartum teaching of a mother who has given birth to her first child. What aspect of teaching is most important? A) Describe the likely reaction of siblings to the new baby. B) Discuss adaptation to grandparenthood by her parents. C) Determine whether father-infant attachment is taking place. D) Assist the mother in identifying the baby's behavior cues

Answer: D Explanation: A) This is not appropriate, because the baby has no siblings. B) Adaptation to grandparenthood is a task for her parents, and not a high priority for teaching to the new mother. C) Although father-infant attachment is important, the mother is the main client, and teaching her directly is a higher priority. D) Helping the mother to identify her baby's behavior cues facilitates the acquaintance phase of maternal-infant attachment.

16)How does the nurse assess for Homans' sign? A) Extending the foot and inquiring about calf pain. B) Extending the leg and inquiring about foot pain. C) Flexing the knee and inquiring about thigh pain. D) Dorsiflexing the foot and inquiring about calf pain.

Answer: D Explanation: A) This is not the correct procedure for assessing for Homans' sign. B) This is not the correct procedure for assessing for Homans' sign. C) This is not the correct procedure for assessing for Homans' sign. D) To assess for thrombophlebitis, the nurse should have the woman stretch her legs out, with the knees slightly flexed and the legs relaxed. The nurse then grasps the foot and dorsiflexes it sharply. If pain is elicited, the nurse notifies the physician/CNM that the woman has a positive Homans' sign. The pain is caused by inflammation of a vessel.

26)The nurse is preparing to assess a newborn's neurological status. Which finding would require an immediate intervention? A) At rest, the infant has partially flexed arms and the legs drawn up to the abdomen. B) When the corner of the mouth is touched, the infant turns the head that direction. C) The infant blinks when the exam light is turned on over the face and body. D) The right arm is flaccid while the infant brings the left arm and fist upward to the head.

Answer: D Explanation: A) This is the normal resting posture of the infant. B) This is the rooting reflex, a normal finding in a newborn. C) Blinking in response to bright lights is an expected finding. D) Asymmetrical movement is not an expected finding, and could indicate neurological abnormality. Muscle tone should be symmetric and diminished muscle tone and flaccidity requires further evaluation.

23)Parents have been told their child has fetal alcohol syndrome (FAS). Which statement by a parent indicates that additional teaching is required? A) "Our baby's heart murmur is from this syndrome." B) "He might be a fussy baby because of this." C) "His face looks like it does due to this problem." D) "Cuddling and rocking will help him stay calm."

Answer: D Explanation: A) Valvular and septal defects are common in babies with FAS. B) FAS babies can be irritable and hyperactive in childhood. C) Facial characteristics of the FAS child include short palpebral fissures, epicanthal folds, broad nasal bridge, flattened midface, short upturned or beaklike nose, micrognathia (abnormally small lower jaw) or hypoplastic maxilla, thin upper lip or vermilion border, and smooth philtrum (groove on upper lip). D) The FASD baby is most comfortable in a quiet, minimally stimulating environment.

26)The nurse is providing care to a pregnant client who experienced a fetal demise in utero. The client opted to wait more than 24 hours prior to scheduling the induction procedure. Which complication is the client at increased risk for based on this data? A) Anxiety B) Hemorrhage C) Hypertension D) Mania

Answer:A Explanation: A) Clients who opt to wait more than 24 hours to schedule the induction procedure after a fetal demise are at an increased risk for anxiety. B) Hemorrhage is not a complication that the client is at increased risk for when waiting more than 24 hours to schedule the induction procedure. C) Hypertension is not a complication that the client is at increased risk for when waiting more than 24 hours to schedule the induction procedure. D) Mania is not a complication that the client is at increased risk for when waiting more than 24 hours to schedule the induction procedure.

24)The nurse will be bringing the parents of a neonate with sepsis to the neonatal intensive care nursery for the first time. Which statement is best? A) "I'll bring you to your baby and then leave so you can have some privacy." B) "Your baby is on a ventilator with 50% oxygen, and has an umbilical line." C) "I am so sorry this has all happened. I know how stressful this can be." D) "Your baby is working hard to breathe and lying quite still, and has an IV."

Answer: D Explanation: A) When bringing parents to see their ill newborn for the first time, it is important to prepare them for what they will see. Bringing parents without preparation is inappropriate. B) Although this statement describes the treatment the baby is receiving, it is worded in medical jargon that will not be understood by most parents. The nurse should describe the equipment being used for the at-risk newborn and its purpose before entering the unit. C) This response focuses on the nurse. Avoid saying "I know how you feel," because it is impossible for the nurse to actually know how clients feel. D) This answer is best because it explains what the parents will see in terminology that they will understand. A trusting relationship is essential for collaborative efforts in caring for the infant. The nurse should respond therapeutically to relate to the parents on a one-to-one basis.

33)Which items will the nurse include in a memory box for a couple who have experienced a stillbirth? Note: Credit will be given only if all correct choices and no incorrect choices are selected. Select all that apply. 1. Pictures 2. ID bracelet 3. Memorycard 4. Bereavementclothing 5. Grief and loss pamphlets

Answer:1, 2, 3, 4 Explanation: When preparing a memory box for a couple who lost their newborn to stillbirth, the nurse will include pictures of the baby, ID bracelets, a memory card, and bereavement clothing. Grief and loss pamphlets should be provided to the couple, but not included in the memory box.

19)It is confirmed that a 14-week fetus has died. The patient has an unfavorable cervix. Which approaches to deliver the fetus should be reviewed with the patient? Note: Credit will be given only if all correct choices and no incorrect choices are selected. Select all that apply. 1. Oxytocin 2. Misoprostol 3. Prostaglandins 4. Laminaria tent 5. Cesareansection

Answer:1, 2, 3, 4 Explanation: Women with an unfavorable cervix may be given vaginal prostaglandin agents, misoprostol, or laminaria tents. Women whose gestations are less than 16 gestational weeks may have a laminaria tent inserted into the cervix before a dilatation and extraction procedure. Women less than 28 weeks' gestation are typically given prostaglandin E2 vaginal suppositories (10-20 mg q 4-6 h), misoprostol 400 mcg vaginally or orally every 4 to 6 hours, and/or oxytocin until spontaneous labor occurs. Cesarean section is not a method identified to deliver the dead fetus.

21)The fetus of a pregnant Latino American patient has died. When assisting the family with the perinatal loss, what should the nurse expect? Note: Credit will be given only if all correct choices and no incorrect choices are selected. Select all that apply. 1. The fetus will be baptized 2. Pictures will be taken of the fetus 3. Cremation will occur within a day 4. The fetus will have an open casket 5. The mother will be dressed in black

Answer:1, 2, 4, 5 Explanation: For a perinatal loss in many Latino American cultures, the fetus will be baptized. Pictures and other mementos are important, as they are utilized to honor the infant in the home. Open casket ceremonies are preferred. The mother may wear traditional dark funeral clothing for an undefined mourning period. Burial is preferred over cremation

20)The nurse is a caring for a patient who is recovering from the loss of her 24-week fetus. Which observations should the nurse identify as being the patient's behavioral responses to this loss? Note: Credit will be given only if all correct choices and no incorrect choices are selected. Select all that apply. 1. Crying 2. Disorientation 3. Time confusion 4. Fear of being alone 5. Absent-mindedness

Answer:1, 4, 5 Explanation: Behavioral responses to loss include fear of being alone, absent-minded behavior, and crying. Cognitive responses to loss include disorientation and time confusion.

23)The nurse is preparing to bathe and dress a fetus that has died. In which order should the nurse provide this postmortem care? 1. Patdry 2. Apply gloves 3. Diaper and dress the fetus 4. Place in second basin of water to rinse 5. Place infant in lukewarm basin of water 6. Bathe according to skin integrity condition

Answer:2, 5, 6, 4, 1, 3 Explanation: When bathing and dressing a fetus as part of postmortem care, the nurse should apply gloves and then place the fetus in a lukewarm basin of water. Cleanse the skin according to the fetus's skin integrity. Place fetus in a second basin of lukewarm water to rinse. Gently pat dry. Diaper and dress fetus in clothing provided.

22)The nurse is preparing to care for a patient who has just learned that her fetus has died. In which order should the nurse adhere to the RESPONDING approach to grieving families? 1. Genuine caring 2. Open communication 3. Nonjudgmental attitude 4. Decision making assistance 5. Spiritual and cultural accommodation 6. Recognition and validation of the loss

Answer:6, 5, 2, 4, 3, 1 Explanation: The nurse should follow the essential elements of responding effectively to the needs of grieving families by following the steps in the acronym RESPONDING: recognition and validation of the loss; spiritual and cultural accommodation; open communication; decision making assistance; nonjudgmental attitude; and genuine caring.

8) The postpartum unit nurse is caring for a client who delivered a term stillborn infant yesterday. The mother is heard screaming at the nutrition services worker, "This food is horrible! You people are incompetent and can't cook a simple edible meal!" The nurse understands this as which of the following? A) An indication that the mother is in the anger phase of grief. B) An abnormal response to the loss of the child. C) Reactive stress management techniques in use. D) Denial of the death of the child she delivered yesterday.

Answer:A Explanation: A) Anger, resulting from feelings of loss, loneliness, and, perhaps, guilt, is a common reaction. Anger may be projected at significant others and/or healthcare team members. B) Anger is a common and normal response to the grief of perinatal loss. C) Reactive stress management techniques do not exist. D) Denial of the loss would manifest as believing that she still has a live fetus in utero.

15)The nurse is supervising care by a new graduate nurse who is working with a couple who have experienced a stillbirth. Which statement made by the new nurse indicates that further instruction is necessary? A) "I should stay out of their room as much as possible." B) "The parents might express their grief differently from each other." C) "My role is to help the family communicate and cope." D) "Hopelessness might be expressed by this family."

Answer:A Explanation: A) Families experiencing perinatal loss need support. The nurse should stay with the couple so they do not feel alone and isolated; however, cues that the couple wants to be alone should be assessed continuously. B) Partners often express grief very differently. C) Families need to communicate to cope effectively. D) Hopelessness is commonly experienced by families after perinatal loss.

24)The nurse educator is teaching a group of students about maternal medical conditions that contribute to stillbirth. Which student statement indicates correct understanding of the material presented? A) "Maternal hypertensive disorders often contribute to stillbirth." B) "Maternal congenital anomalies contribute to stillbirth." C) "Maternal chromosomal abnormalities contribute to stillbirth." D) "Maternal malformations contribute to stillbirth."

Answer:A Explanation: A) Maternal hypertensive disorders are a contributing factor to stillbirth. This statement indicates correct understanding by the student. B) Fetal, not maternal, congenital anomalies contribute to stillbirth. This statement indicates the need for additional education. C) Fetal, not maternal, chromosomal abnormalities contribute to stillbirth. This statement indicates the need for additional education. D) Fetal, not maternal, malformations contribute to stillbirth. This statement indicates the need for additional education.

11)The nurse is working with a family who experienced the stillbirth of a son 2 months ago. Which statement by the mother would be expected? A) "I seem to keep crying for no reason." B) "The death of my son hasn't changed my life." C) "I have not visited my son's gravesite." D) "I feel happy all the time."

Answer:A Explanation: A) Mourning may be manifested by certain behaviors and rituals, such as weeping, which help the person experience, accept, and adjust to the loss. B) A fetal loss is devastating to parents, and results in significant life changes. C) Mourning may be manifested by certain behaviors and rituals, such as visiting a gravesite, which can help the person experience, accept, and adjust to the loss. D) Happiness is not an expected part of mourning.

14)The labor and delivery nurse is caring for a client whose labor is being induced due to fetal death in utero at 35 weeks' gestation. In planning intrapartum care for this client, which nursing diagnosis is most likely to be applied? A) Powerlessness B) Urinary Elimination, Impaired C) Coping: Family, Readiness for Enhanced D) Skin Integrity, Impaired

Answer:A Explanation: A) Powerlessness is commonly experienced by families who face fetal loss. Powerlessness is related to a sense of lack of control in the current situational crisis. B) Impaired urinary elimination is not related to fetal loss. C) Ineffective family coping is more likely to occur at this point than readiness for enhanced family coping. D) Impaired skin integrity is not related to fetal loss.

13)The nurse is present when a mother and her partner are told that their 35-week fetus has died. Which nursing intervention should the nurse perform first? A) Encourage open communication with the family and the healthcare team. B) Ask the family to withhold questions until the next day. C) Request that another nurse come and care for this family. D) Contact a local funeral home to help the family with funeral plans.

Answer:A Explanation: A) The top priority for the nurse is to encourage open communication. The nurse functions as an advocate for the family in organizing interdisciplinary involvement, maintaining continuity of care, offering the opportunity for open communication, and ensuring that the family's wishes regarding their loss experience are honored. B) Requesting that the couple not ask questions does not facilitate communication. The nurse should provide ongoing opportunities for the couple to ask questions. C) Requesting that another nurse care for the family does not facilitate communication. When possible, the same nurse should provide care for the couple so that a therapeutic relationship can be established. D) Funeral plans can be made later and are usually made by the family.

5) A couple requests to see their stillborn infant. How should the nurse prepare the infant? Note: Credit will be given only if all correct choices and no incorrect choices are selected. Select all that apply. A) Wrapping the infant in a blanket B) Removing all blankets from the infant C) Placing a hat on the infant D) Removing any identification from the infant E) Placing a diaper on the infant is not necessary.

Answer:A, C Explanation: A) After bathing, the infant should be placed in a suitable-sized gown and then wrapped in a blanket. Many parents will eventually remove the covering to inspect the infant; however, applying a covering allows them time to adjust to the appearance at their own pace. B) Removing blankets from the infant would not be appropriate. C) A hat can be applied to cover birth defects. This allows the parents an opportunity to view the infant before seeing the birth defect. D) Removing any identification from the infant would not be appropriate

18)The community nurse is planning care for a family that experienced the loss of twins at 20 weeks. Which of these steps should be part of the nurse's care of this family? A) Base care on the reactions of previous clients who experienced stillbirth. B) Express the belief that the family will be able to get through this experience. C) Encourage the couple to keep their feelings to themselves. D) Honor the birth by reminding the couple that their babies are happy in heaven

Answer:B Explanation: A) Care for a family experiencing perinatal loss must be individualized; no two families will respond in the same way. B) Maintaining belief is defined as believing in the parents' capacity to get through the event and face a future with meaning, and it is one of the attributes of caring theory. C) Communication should be encouraged. D) Clichés are to be avoided.

35)The nurse is assessing a client's risk for death during the postpartum period. Which finding would cause the nurse to increase surveillance for this client? A) Maternal age of 25 B) History of bipolar disorder C) Increased body mass index D) Decreased blood pressure

Answer:C Explanation: A) Advanced maternal age is a risk factor for maternal death during the postpartum period. Advanced maternal age is defined as greater than 35 years of age. B) A history of bipolar disorder increases the risk of postpartum depression and psychosis, not maternal demise. C) An increased body mass index is often a risk factor for maternal demise during the postpartum period. D) An increased, not decreased, blood pressure is a risk factor for maternal demise during the postpartum period.

4) The nurse is anticipating the arrival of a couple in the labor unit. It has been determined that the 37-week fetus has died in utero from unknown causes. What should the nurse include in the plan of care for this couple? A) Allow the couple to adjust to the labor unit in the waiting area. B) Place the couple in a labor room at the end of the hall with an empty room next door. C) Encourage the father to go home and rest for a few hours. D) Contact the mother's emergency contact person and explain the situation.

Answer:B Explanation: A) Care should be taken not to leave the couple in the waiting room with other expectant parents or visitors waiting for news from other women in labor. B) Upon arrival at the facility, the couple with a known or suspected fetal demise should immediately be placed in a private room. When possible, the woman should be in the room that is farthest away from other laboring women. C) The couple should be allowed to remain together as much as they wish. Provide privacy as needed and maintain a supportive environment. D) Some couples may want outside support, such as family members or friends, to be present during the labor. The nurse needs to facilitate the couple's wishes, including not contacting anyone if that wish is expressed.

9) The nurse is caring for a client who experienced the birth of a stillborn son earlier in the day. The client is from a culture where a woman's status is dominated by themes of motherhood and childrearing. What behavior would the nurse expect in this client? A) Crying inconsolably B) Expressing feelings of failure as a woman C) Requesting family members to be present D) Showing little emotion

Answer:B Explanation: A) Inconsolable crying is not a sign of a maternalistic culture, but is a universal sign of deep grief. B) Mothers will often blame themselves, whether by commission or omission, particularly in cultures where a woman's status is dominated by themes of motherhood and childrearing. C) Inconsolable crying is not a sign of a maternalistic culture, nor is the presence of multiple family members. D) Showing little emotion indicates a stoic culture, which might not be maternalistic.

3) The community nurse has identified that the mother who gave birth to a stillborn baby last week is an intuitive griever. Which behavior has the nurse encountered that would lead to this assessment? A) The mother verbalized that her problem-solving skills have been helpful during this process. B) The mother repeatedly talks about her thoughts, feelings, and emotions about losing her child. C) The mother talks little about her experience, and appears detached and unaffected by the loss of her child. D) The mother has asked close friends, co-workers, and relatives not to call or visit

Answer:B Explanation: A) Instrumental grievers tend to use problem-solving skills in the grief process. B) Intuitive grievers tend to feel their way through the loss and seek emotional and psychosocial support. C) Instrumental grievers can appear detached and unaffected by the experience. D) Intuitive grievers seek emotional and psychosocial support from friends and family.

16)The nurse is working with a laboring woman who has a known intrauterine fetal demise. To facilitate the family's acceptance of the fetal loss, after delivery the nurse should do which of the following? A) Encourage the parents to look at the infant from across the room. B) Offer the parents the choice of holding the infant in their arms. C) Take the infant to the morgue immediately. D) Call family members and inform them of the birth

Answer:B Explanation: A) Looking at the infant from a distance would not be helpful. B) The nurse should offer the couple the opportunity to see and hold the infant and reassure the couple that any decision they make for themselves is the right one. C) Taking the infant to the morgue before the parents have a chance to see and hold it would not be appropriate. D) Calling family members would not be helpful or appropriate for the nurse to do.

1) The nurse is planning an in-service presentation about perinatal loss. Which statements should the nurse include in this presentation? Note: Credit will be given only if all correct choices and no incorrect choices are selected. Select all that apply. A) "Perinatal loss refers to third-trimester fetal death in utero." B) "Perinatal loss occurs more frequently in assisted reproduction." C) "Perinatal loss rates have declined in the United States over the past few years." D) "Perinatal loss includes 25% of stillbirths occurring before the onset of labor." E) "Perinatal loss rarely causes an emotional problem for the family."

Answer:B, C Explanation: A) Perinatal loss is the death of a fetus or infant from the time of conception through the end of the newborn period 28 days after birth. B) Pregnancies conceived by in vitro fertilization have higher rates of pregnancy loss and pregnancy complications. C) Perinatal loss in industrialized countries has declined in recent years as early diagnosis of congenital anomalies and advances in genetic testing techniques have increased the use of elective termination. D) It is estimated that 50% of stillbirths occur before the onset of labor, with more than 24.4% occurring between 20 and 28 weeks' gestation. E) Perinatal losses are grieved by most couples and families like any other death. Like other grief, the grief over perinatal loss can become overwhelming.

29)The nurse is providing care to a Hispanic client who has experienced a stillbirth. Which action by the nurse is appropriate when providing culturally competent care to the client and family members? A) Asking the paternal grandfather if they will sit in shiva B) Asking the oldest male member of the family if he would like to participate in the cremation C) Asking the parents if they wish to have their baby baptized D) Asking the parents if they wish to participate in the ritual of painting the baby's body

Answer:C Explanation: A) A Jewish, not Hispanic, family may sit in shiva after the loss of a baby. This action is not appropriate when providing culturally competent care to the client and family. B) A Hindu, not Hispanic, family, may want the oldest male member of the family to participate in the baby's cremation. This action is not appropriate when providing culturally competent care to the client and family. C) Many Hispanic families will want the baby to be baptized. This action is appropriate when providing culturally competent care to the client and family. D) A Native American, not Hispanic, family may want to participate in a ritual of painting the baby's body. This action is not appropriate when providing culturally competent care to the client and family.

34)The nurse is providing care to a couple who have experienced a perinatal loss. Which nursing diagnosis is appropriate to include in the plan of care? A) Acute Pain B) Complicated Grieving C) Powerlessness D) Insomnia

Answer:C Explanation: A) Acute pain is not an appropriate nursing diagnosis to include in the couple's plan of care. B) Complicated grieving is not an appropriate nursing diagnosis to include in the couple's plan of care. C) Powerlessness related to the sense of a lack of control in the current situational crisis is an appropriate nursing diagnosis to include in the couple's plan of care. D) Insomnia is not an appropriate nursing diagnosis to include in the couple's plan of care.

2) The nurse has returned from working as a maternal-child nurse volunteer for a non- governmental organization. After completing a community presentation about this experience, the nurse knows that learning has occurred when a participant states which of the following? A) "Malaria is a chronic disease, and rarely causes fetal loss." B) "Escherichia coli bacteria can cause diarrhea, but not stillbirth." C) "Group B streptococci can cause infection and the death of the fetus." D) "Viral infections don't cause fetal death in developing nations

Answer:C Explanation: A) Malaria is associated with a high stillbirth rate when contracted for the first time by the mother during the pregnancy, and it carries a much higher morbidity and mortality rate in developing countries. B) E. coli can cause ascending infections prior to or after rupture of membranes. C) Group B streptococci can cause ascending infections prior to or after rupture of membranes. D) Viral infections play a significant role in fetal death in developing countries

32)Which nursing action is appropriate prior to presenting the stillborn newborn to the parents for holding? A) Staying with the family while they hold and say goodbye to their newborn B) Telling the family that they should not hold their baby because it will be too painful C) Preparing the family by telling them what they will see, feel, and smell D) Telling the family that they can have up to thirty minutes with their baby

Answer:C Explanation: A) The nurse should offer the family time alone with the baby, but check back frequently within previously specified time frames. B) The nurse should not impose his or her beliefs on the couple when providing care. C) The nurse should prepare the family for holding the baby by telling them what they will see, feel, and smell. D) The nurse should allow the family to have as much time as they need to hold their baby and say goodbye.

27)A couple who experienced a fetal demise would like to determine the specific cause. Which method of fetal testing will the nurse prepare for based on this request? A) Diabetes testing B) Urine toxicology testing C) Infectious disease studies D) Chromosomal

Answer:D Explanation: A) Diabetes testing is a method of maternal testing, not fetal testing, to determine the cause of fetal demise. B) Urine toxicology testing is a method of maternal testing, not fetal testing, to determine the cause of fetal demise. C) Infectious disease studies are a method of maternal testing, not fetal testing, to determine the cause of fetal demise. D) Chromosomal studies are a method of fetal testing that is used to determine the cause of fetal demise.

12)A 15-year-old client has delivered a 22-week stillborn fetus. What does the nurse understand? A) Grieving a fetal loss manifests with very similar behaviors regardless of the age of the client. B) Teens tend to withhold emotions and need older adults with the same type of loss to help process the experience. C) Most teens have had a great deal of contact with death and loss and have an established method of coping. D) Assisting the client might be difficult because of her mistrust of authority figures

Answer:D Explanation: A) Grieving behaviors are very different in adolescents than in older clients. Adolescents rely heavily on peer support. B) Adolescents rely more heavily on peer support than on adults for emotional support. C) For many adolescents, a pregnancy loss may be the first time they are experiencing the psychologic impact of grief. D) Adolescents rely heavily on peer support and have a natural mistrust of authority figures, which can make assisting them more difficult.

31)The nurse is preparing a stillborn newborn to be held by the parents. Which action by the nurse is appropriate? A) Using baby lotion so that the newborn smells like a baby B) Placing cotton in the newborn's nostrils if seepage is noted C) Using a washcloth when bathing the newborn's skin to decrease tissue sloughing D) Placing the newborn in the provided outfit and blanket

Answer:D Explanation: A) Scented baby products should be avoided, as they will enhance skin maceration. B) Petroleum jelly, not cotton, should be placed in the newborn's nostrils if seepage is noted. C) A gloved hand, not a washcloth, should be used when bathing the newborn's skin to decrease tissue sloughing. D) The baby should be placed in the provided outfit and wrapped in a blanket in preparation for parents who wish to hold their baby after death.

30)The nurse is providing care to a couple who have experienced a stillbirth. Which is a therapeutic statement from the nurse when providing care to the client? A) "You are young; you can try again." B) "It's a blessing in disguise." C) "God needed another angel in heaven." D) "I am here to listen if you would like to talk about it."

Answer:D Explanation: A) Telling the couple that they are young and can try again diminishes their current pain. B) Telling the couple that it was a blessing in disguise often unfortunately occurs when there are fetal deformities. There are no adequate explanations as to why the couple is experiencing this tragedy, and none should be offered. C) It is not appropriate for the nurse to impose his or her belief system on the family. D) This statement reassures the couple that the nurse cares and is a therapeutic statement from the

17)The mother of a client who has experienced a term stillbirth arrives at the hospital and goes to the nurses' desk. The mother asks what she should say to her daughter in this difficult time. What is the nurse's best response? Note: Credit will be given only if all correct choices and no incorrect choices are selected. Select all that apply. A) "Use clichés; your daughter will find the repetition comforting." B) "Remind her that she is young and can have more children." C) "Keep talking about other things to keep her mind off the loss." D) "Express your sadness, and sit silently with her if she doesn't respond." E) "Encourage her to talk about the baby whenever she wants to."

Answer:D, E Explanation: A) Although grieving people often say that it is easier to forgive less-than-helpful remarks than to forgive those who do not reach out to them at all, the use of clichés can be harmful. B) Reminding the mother that she is young enough to have other children downplays the importance of this baby. C) Nonstop talking does not allow the client to express her emotions and feelings. Sitting in silence with a grieving family is often the most helpful form of intervention available. D) Silence is commonly what is needed most, and simply saying "I'm sorry for your loss" might help to facilitate communication. E) Talking is a way for the client experiencing grief and begin to come to terms with what has happened, and is important for resolution of grief. Intuitive grievers will need to talk about the event.


Related study sets

Chapter 24: Newborn Nutrition and Feeding Perry: Maternal Child Nursing Care, 6th Edition, Chapter 23: Nursing Care of the Newborn and Family Perry: Maternal Child Nursing Care, 6th Edition, Chapter 22: Physiologic and Behavioral Adaptations of the N...

View Set

Econ 110: Spring; Al Hamdi (Exam 2 HWs)

View Set

Joey's handy dandy econ final study guide part 3, Ch. 16-17

View Set

Vector Practice--Dot Product and Addition

View Set

Prep U:40: Fluid, Electrolyte, and Acid-Base Balance

View Set